MỘT SỐ ĐỊNH LÝ CƠ SỞ TRONG LÝ THUYẾT SỐ GIẢI TÍCH

117 697 7
MỘT SỐ ĐỊNH LÝ CƠ SỞ TRONG LÝ THUYẾT SỐ GIẢI TÍCH

Đang tải... (xem toàn văn)

Tài liệu hạn chế xem trước, để xem đầy đủ mời bạn chọn Tải xuống

Thông tin tài liệu

Chương trình lý thuyết số hè 2012 ở Viện nghiên cứu cao cấp về toán được chia thành hai hoạt động. Một hoạt động là lớp học những định lý kinh điển của lý thuyết số giải tích. Tài liệu học là cuốn sách Nhập môn lý thuyết số giải tích của Chandrasekharan. Song song với lớp học là sinh hoạt chuyên đề xung quanh công trình mới của BhargavaShankar về hạng trung bình của đường cong elliptic. Trong lớp học lý thuyết số giải tích, học viên được giao đọc và trình bày một chương trong cuốn sách của Chandrashkharan. Cuốn kỷ yếu này tập hợp lại ghi chép của học viên khi đọc sách. Cuốn kỷ yếu có thể rất hữu ích cho sinh viên khoa toán, đặc biệt là sinh viên các trường Sư phạm. Những định lý trình bày ở đây khá gần gũi với lý thuyết số sơ cấp. Nếu đã nắm vững lý thuyết số sơ cấp, để theo dõi những nội dung trình bày ở đây, bạn đọc cần nắm vững một vài kiến thức cơ bản về hàm một biến phức. Những người tham gia Chương trình lý thuyết số hè 2012 đã làm việc với một tinh thần rất nghiêm túc. Điều đó được thể hiện qua chất lượng của những ghi chép được tập hợp lại ở đây. Cảm ơn các bạn sinh viên Hà nội, Huế, thành phố Hồ Chí Minh và cả từ nửa kia của trái đất, đã không quản đường xa đến Viện nghiên cứu cao cấp để học hè. Đặc biệt cảm ơn Tiến sĩ Bùi Mạnh Hùng, người đã soi sáng chúng tôi bằng những hiểu biết sâu về lý thuyết số giải tích của anh và Thạc sĩ Trần Quang Hoá, người đã giúp tôi biên tập cuốn kỷ yếu này

MỘT SỐ ĐỊNH LÝ CƠ SỞ TRONG LÝ THUYẾT SỐ GIẢI TÍCH Kỷ yếu lớp hè 2012 của GS. Ngô Bảo Châu HÀ NỘI, THÁNG 9 NĂM 2012 Mục lục Lời nói đầu i 1 Biểu diễn số nguyên dương thành tổng của các bình phương 1 2 Luật thuận nghịch bậc hai 10 3 Một chứng minh cho luật thuận nghịch bậc hai 21 4 Một chứng minh sơ cấp cho luật thuận nghịch bậc hai 26 5 Hàm số học và điểm nguyên 30 6 Định lý Chebyshev về sự phân bố số nguyên tố 47 7 Định lý Weyl về phân bố đều và định lý Kronecker 67 8 Định lí Minkowski về điểm nguyên trong một tập lồi 77 9 Định lí Dirichlet về số nguyên tố trong một cấp số cộng 86 10 Định lí số nguyên tố 107 Lời nói đầu Chương trình lý thuyết số hè 2012 ở Viện nghiên cứu cao cấp về toán được chia thành hai hoạt động. Một hoạt động là lớp học những định lý kinh điển của lý thuyết số giải tích. Tài liệu học là cuốn sách "Nhập môn lý thuyết số giải tích" của Chandrasekharan. Song song với lớp học là sinh hoạt chuyên đề xung quanh công trình mới của Bhargava-Shankar về hạng trung bình của đường cong elliptic. Trong lớp học lý thuyết số giải tích, học viên được giao đọc và trình bày một chương trong cuốn sách của Chandrashkharan. Cuốn kỷ yếu này tập hợp lại ghi chép của học viên khi đọc sách. Cuốn kỷ yếu có thể rất hữu ích cho sinh viên khoa toán, đặc biệt là sinh viên các trường Sư phạm. Những định lý trình bày ở đây khá gần gũi với lý thuyết số sơ cấp. Nếu đã nắm vững lý thuyết số sơ cấp, để theo dõi những nội dung trình bày ở đây, bạn đọc cần nắm vững một vài kiến thức cơ bản về hàm một biến phức. Những người tham gia Chương trình lý thuyết số hè 2012 đã làm việc với một tinh thần rất nghiêm túc. Điều đó được thể hiện qua chất lượng của những ghi chép được tập hợp lại ở đây. Cảm ơn các bạn sinh viên Hà nội, Huế, thành phố Hồ Chí Minh và cả từ nửa kia của trái đất, đã không quản đường xa đến Viện nghiên cứu cao cấp để học hè. Đặc biệt cảm ơn Tiến sĩ Bùi Mạnh Hùng, người đã soi sáng chúng tôi bằng những hiểu biết sâu về lý thuyết số giải tích của anh và Thạc sĩ Trần Quang Hoá, người đã giúp tôi biên tập cuốn kỷ yếu này. Tháng 9, 2012 i Biểu diễn số nguyên dương thành tổng của các bình phương Người trình bày: Trần Quang Hóa 1 Mở đầu Phần này tôi sẽ trình bày bài toán biểu diễn một số tự nhiên thành tổng của các bình phương. Chúng ta xét bài toán sau: cho k ∈ Z và k ≥ 1, xét tập hợp Sk = {n ∈ N | n = x21 + x22 + · · · + x2k , xi ∈ Z, ∀i = 1, 2, . . . , k}. Phần này trình bày chủ yếu dựa vào chương IV [1], chương X [2] và chương XI [3]. Nội dung chính của chương này là chứng minh định lý Euler về biểu diễn số nguyên tố dạng 4k + 1 là tổng bình phương của hai số nguyên và định lý Lagrange nói rằng mọi số nguyên dương đều là tổng bình phương của 4 số nguyên. Trước khi trình bày, chúng ta cần sơ lược về lý thuyết thặng dư. Nó là một trong những vấn đề quan trọng của lý thuyết số,là công cụ chính để giải phương trình bậc hai ax2 + bx + c ≡ 0(mod m), (a, m) = 1. (1) Vấn đề này quy về việc giải phương trình ax2 + bx + c ≡ 0(mod p), với p là số nguyên tố lẻ và (a, p) = 1. Do p lẻ nên (p, 4) = 1, vậy phương trình tương đương với (2ax + b)2 ≡ (b2 − 4ac)(mod p). Vậy thực sự ta chỉ cần xét phương trình x2 ≡ a(mod p), (a, p) = 1. (2) Trước hết, ta xét ký hiệu Legendre, công cụ cho ta biết khi nào phương trình (2) có nghiệm. Trong chương tiếp theo, việc sử dụng luật thuận nghịch bậc hai sẽ cho ta câu trả lời hoàn toàn khi nào phương trình (2) có nghiệm. 1 2 Ký hiệu Legendre Cho p là số nguyên tố lẻ và a ∈ Z sao cho (a, p) = 1. Ta nói a thặng dư bậc hai theo modulo p nếu phương trình x2 ≡ a(mod p) có nghiệm, ký hiệu aRp. Ngược lại, ta nói a bất thặng dư bậc hai theo modulo p và ký hiệu aN p. thặng dư bậc Nhận xét: Trong các số nguyên 1, 2, 3, . . . , p − 1, có đúng p−1 2 p−1 hai theo modulo p và có đúng 2 bất thặng dư bậc hai theo modulo p. Bây giờ, cho p là số nguyên tố lẻ và m là số nguyên sao cho (m, p) = 1. Ta định nghĩa ký kiệu Legendre m = p +1, nếu mRp, −1, nếu mN p. Chúng ta mở rộng định nghĩa ký hiệu Legendre m = 0, nếu p|m. p Từ nhận xét trên, chúng ta có p−1 m=0 m = 0. p Tiếp theo, ta sẽ nêu ra một số tích chất cơ bản của ký hiệu Legendre. Tính chất 1. Nếu m1 ≡ m2 (mod p), thì m1 p = m2 . p Tính chất 2 (Tiêu chuẩn Euler). Nếu p là một số nguyên tố lẻ và a là một số nguyên. Khi đó 1 a 2 (p−1) ≡ 1(mod p) nếu và chỉ nếu a là một thặng dư bậc hai theo modulo p. Chứng minh. Nếu a là thặng dư bậc hai theo modulo p thì tồn tại x0 ∈ Z sao cho x20 ≡ a(mod p). Vì (a, p) = 1 nên (x0 , p) = 1. Do p lẻ nên 12 (p − 1) ∈ Z, chúng ta thu được 1 ≡ a 2 (p−1) (mod p). xp−1 0 1 Theo định lý Fermat xp−1 ≡ 1(mod p). Do đó a 2 (p−1) ≡ 1(mod p). 0 2 1 Ngược lại, phương trình x 2 (p−1) ≡ 1(mod p) có tối đa 21 (p − 1) nghiệm. Và theo nhận xét trên có đúng 21 (p − 1) phần tử thặng dư bậc hai theo modulo 1 p và chúng đều là nghiệm của phương trình x 2 (p−1) ≡ 1(mod p). Do đó, nếu 1 a bất thặng dư bậc hai theo modulo p thì a 2 (p−1) ≡ 1(mod p). Tính chất 3. Nếu p là một số nguyên tố lẻ và a là một số nguyên sao cho (a, p) = 1. Khi đó 1 a a 2 (p−1) ≡ (mod p). p Chứng minh. Do p là số nguyên tố lẻ và (a, p) = 1, theo định lý Fermat ta có p−1 p−1 ap−1 − 1 = a 2 − 1 a 2 + 1 ≡ 0(mod p). Do đó a p−1 2 ≡ 1(mod p), (3) ≡ −1(mod p). (4) hoặc a p−1 2 Do đó, nếu a bất thặng dư bậc hai, nó không thỏa mãn (3) nên nó thỏa mãn (4). Theo định nghĩa của ký hiệu Legendre, ta có điều phải chứng minh. Tính chất 4. Với mọi số nguyên m, n ta có m p n mn = . p p Tính chất 5. Cho p là số nguyên tố lẻ, khi đó 1 −1 = (−1) 2 (p−1) . p Hơn nữa, −1Rp ⇔ p ≡ 1(mod 4) và −1N p ⇔ p ≡ 3(mod 4). 3 Tổng của hai bình phương Xét tập S2 = {n ∈ N | n = x21 + x22 , x1 , x2 ∈ Z}. Ví dụ 0 ∈ S2 vì 0 = 02 + 02 , 1 ∈ S2 vì 1 = 02 + 12 , 2 ∈ S2 vì 2 = 12 + 12 . 3 Bổ đề 1. Tập S2 đóng đối với phép nhân, nghĩa là nếu a, b ∈ S2 thì ab ∈ S2 . Chứng minh. Ta có đồng nhất thức (x21 + y12 )(x22 + y22 ) = (x1 x2 + y1 y2 )2 + (x1 y2 − x2 y1 )2 . Bổ đề 2. Nếu p là số nguyên tố dạng 4k + 1, thì tồn tại số nguyên x sao cho x2 + 1 = mp, với 0 < m < p. Chứng minh. Do p ≡ 1(mod 4) nên −1 là thặng dư bậc hai của p, khi đó tồn tại một số nguyên x ∈ {1, 2, . . . , 21 (p − 1)}, sao cho x2 ≡ −1(mod p). Suy ra tồn tại số nguyên m sao cho x2 + 1 = mp. Do x < p/2 nên x2 + 1 < (p/2)2 + 1 < p2 . Do đó x2 + 1 = mp, với 0 < m < p. Định lý 1 (Euler). Mọi số nguyên tố dạng 4k + 1 đều được biểu diễn thành tổng bình phương của hai số nguyên. Chứng minh. Xét tập Ω = {m ∈ Z | mp = x21 +x22 , 0 < m < p, xi ∈ Z, x1 , x2 không đồng thời chia hết cho m}. Khi đó, theo Bổ đề 2, Ω = ∅ nên theo Bổ đề Zorn, tồn tại phần tử nhỏ nhất là m ∈ Ω. Ta chứng minh m = 1. Giả sử m > 1. Vì m ∈ Ω nên mp = a21 + a22 , a1 , a2 ∈ Z . Lấy a2 ≡ a1 (mod m), b2 ≡ b1 (mod m) sao cho |a2 |, |b2 | ≤ m2 . Từ a22 + b22 ≡ a21 + b21 ≡ 0(mod m) nên a22 + b22 = sp, s ∈ Z . 2 Do |a2 |, |b2 | ≤ m2 nên a22 + b22 ≤ 2( m2 )2 = m2 < m2 . Vậy s < m. Mặt khác, s > 0 vì nếu s = 0 thì a2 = b2 = 0. Do đó a1 ≡ b1 ≡ 0(mod m). Điều này mâu thuẫn với cách chọn m. Vậy 0 < s < m. Chúng ta có m2 sp = (a21 + b21 )(a22 + b22 ) = (a1 a2 + b1 b2 )2 + (a1 b2 − a2 b1 )2 , trong đó a1 a2 + b1 b2 ≡ a22 + b22 ≡ 0(mod m), a1 b2 − a2 b1 ≡ a2 b2 − a2 b2 ≡ 0(mod m). 4 (5) Vậy từ (5) suy ra ( a1 a2 + b 1 b 2 2 a1 b2 − a2 b1 2 ) +( ) = sp, m m trong đó 0 < s < m. Điều này mâu thuẫn với tính nhỏ nhất của m. Vậy m = 1 và ta có điều phải chứng minh. Tiếp theo chúng ta đưa ra điều kiện để một số nguyên dương biểu diễn được thành tổng của hai bình phương. Ta nói biểu diễn n = x2 +y 2 là nguyên thủy nếu (x, y) = 1. Bổ đề 3. Nếu n chia hết bởi số nguyên tố p dạng 4k + 3, thì n không có biểu diễn nguyên thủy. Chứng minh. Giả sử n = x2 + y 2 , với (x, y) = 1. Khi đó p | (x2 + y 2 ) và p x, p y. Từ (p, x) = 1 nên tồn tại số nguyên m sao cho mx ≡ y(mod p). Do đó x2 + (mx)2 ≡ x2 + y 2 ≡ 0(mod p). Suy ra p | x2 (1+m2 ), và do p x nên p | (1+m2 ). Vậy m2 ≡ −1(mod p) hay -1 là thặng dư bậc hai theo modulo p. Điều này không thể vì p ≡ 3(mod 4). Bổ đề 4. Nếu p là số nguyên tố p dạng 4k + 3 và c là số nguyên lẻ sao cho pc | n và pc+1 n, thì n không thể biểu diễn thành tổng của hai bình phương. Chứng minh. Giả sử n = x2 + y 2 , theo bổ đề 3 ta có (x, y) = d > 1. Ta viết x = dX, y = dY, (X, Y ) = 1 và n = d2 (X 2 + Y 2 ) = d2 N. Gọi pr là lũy thừa cao nhất của p sao cho pr | d. Lúc đó pc−2r | N. Vì c lẻ nên c − 2r > 0. Vậy ta tìm được số nguyên N sao cho N = X 2 + Y 2 , (X, Y ) = 1, p | N với p ≡ 3(mod 4). Điều này mâu thuẫn với Bổ đề 3. Vậy Bổ đề 4 được chứng minh. Định lý 2. Số nguyên dương n biểu diễn thành tổng hai bình phương khi và chỉ khi mọi ước nguyên tố của nó dạng 4k + 3 có lũy thừa chẵn trong phân tích tiêu chuẩn của n. Chứng minh. • Điều kiện cần: Suy ra từ Bổ đề 4. • Điệu kiện đủ: Ta có thể viết n = n21 n2 , với n2 không có ước nguyên tố dạng 4k + 3. Do đó n2 chỉ có ước nguyên tố là 2 hoặc p dạng 4k + 1. Do 2 và p đều có thể biễu diễn thành tổng hai bình phương. Theo Bổ đề 1, ta có n2 = a2 + b2 . Từ đó n = (n1 a)2 + (n1 b)2 . Vậy định lý được chứng minh hoàn toàn. 5 4 Tổng của ba bình phương Bây giờ ta xét tập S3 = {n ∈ N∗ | n = x21 + x22 + x23 , x1 , x2 , x3 ∈ Z}. Bổ đề 5. Nếu n ∈ S3 thì n ≡ 7(mod 8). Chứng minh. Do x2 ≡ 0, 1, 4(mod 8) nên n = x21 + x22 + x23 ≡ 7(mod 8). Việc xác định tập S3 khó khăn hơn S2 bởi vì tập S3 không đóng đối với phép nhân. Chẳng hạn, 3 = 12 + 12 + 12 và 5 = 22 + 12 + 02 nên 3, 5 ∈ S3 nhưng 15 ∈ S3 vì 15 ≡ 7(mod 8). Bổ đề 6. Nếu n ∈ S3 và 4 | n thì n 4 ∈ S3 . Chứng minh. Giả sử n = x21 + x22 + x23 . Vì x2 ≡ 0, 1(mod 4) và n = x21 + x22 + x23 ≡ 0(mod 4) nên suy ra x2i ≡ 0(mod 4), ∀i = 1, 2, 3. Do đó xi ≡ 0(mod 2), ∀i = 1, 2, 3. Vậy n x1 x2 x3 = ( )2 + ( )2 + ( )2 , 4 2 2 2 n hay 4 ∈ S3 . Giả sử số nguyên dương n có dạng n = 4e (8k + 7). Nếu n ∈ S3 thì theo Bổ đề 6, 4ne ∈ S3 hay n = 8k + 7 ∈ S3 . Điều này mâu thuẫn với Bổ đề 5. Vậy n = 4e (8k + 7) ∈ S3 . Điều ngược lại cũng đúng và được phát biểu như sau. Định lý 3 (Gauss). n ∈ S3 ⇐⇒ n = 4e (8k + 7). Người đầu tiên chứng minh điều kiện cần là Gauss. Sau đó Lejeune Dirichlet và Landau đã đơn giản chứng minh của Gauss. Sau này N.C. Ankeny đã chứng minh Định lý Gauss khá "cơ bản". Chứng minh của Ankeny dựa vào định lý Minkowski về số điểm nguyên chứa trong tập lồi (xem chương IX [1]) và định lý Dirichlet về dãy số nguyên tố trong cấp số cộng (xem chương X [1]). Chứng minh này vượt quá mục đích của chương này nên không được đưa ra ở đây. Bạn đọc tham khảo A. Schinzel [4]. 6 5 Tổng của bốn bình phương Bây giờ ta xét tập S4 = {n ∈ N∗ | n = x21 + x22 + x23 + x24 , x1 , x2 , x3 , x4 ∈ Z}. Bổ đề 7. Tập S4 đóng đối với phép nhân. Chứng minh. Từ đồng nhất thức (x21 + x22 + x23 + x24 )(y12 + y22 + y32 + y42 ) = z12 + z22 + z32 + z42 , (6) trong đó z1 = x1 y1 + x2 y2 + x3 y3 + x4 y4 , z2 = x1 y2 − x2 y1 + x3 y4 − x4 y3 , z3 = x1 y3 − x3 y1 + x4 y2 − x2 y4 , z4 = x1 y4 − x4 y1 + x2 y3 − x3 y2 , chỉ ra rằng tích của hai số nguyên biễu diễn được thành tổng của bốn bình phương là số nguyên biểu diễn được thành tổng của bốn bình phương. Bổ đề 8. Nếu p là số nguyên tố lẻ, thì tồn tại hai số nguyên x, y sao cho 1 + x2 + y 2 = mp, với 0 < m < p. Chứng minh. Với mọi 0 ≤ x = y ≤ 12 (p − 1) ta có x2 ≡ y 2 (mod p) và −1 − x2 ≡ −1 − y 2 (mod p). . Do Do đó #{x2 | 0 ≤ x ≤ 12 (p − 1)} = #{−1 − y 2 | 0 ≤ y ≤ 12 (p − 1)} = p+1 2 1 chỉ có p lớp thặng dư theo modulo p nên tồn tại 0 ≤ x, y ≤ 2 (p − 1) sao cho x2 ≡ (−1 − y 2 )(mod p) ⇐⇒ 1 + x2 + y 2 = mp. Từ 0 ≤ x, y ≤ 21 (p − 1), ta có 1 1 + x2 + y 2 < 1 + 2( p)2 < p2 , 2 do đó 1 + x2 + y 2 = mp, 0 < m < p. Định lý 4 (Lagrange). Mọi số nguyên dương n đều biểu diễn được thành tổng của bốn bình phương. 7 Chứng minh. Ta có 1 ∈ S4 vì 1 = 12 + 02 + 02 + 02 nên ta giả sử n > 1. Do n > 1 là tích của các số nguyên tố, và 2 = 12 + 12 + 02 + 02 nên 2 ∈ S4 . Do S4 đóng với phép nhân nên để chứng minh định lý ta chỉ cần chứng minh mọi số nguyên tố lẻ p đều biễu diễn được thành tổng của bốn bình phương. Xét tập . Ω = {m ∈ Z | mp = x21 +x22 +x23 +x24 , 0 < m < p, xi ∈ Z, ∃xi ..m, i = 1, . . . , 4}. Khi đó, theo Bổ đề 8, Ω = ∅ nên theo Bổ đề Zorn, tồn tại phần tử nhỏ nhất là m0 ∈ Ω, tức là m0 p = x21 + x22 + x23 + x24 , 0 < m0 < p. (7) Ta chứng minh m0 = 1. Giả sử m0 > 1. Đầu tiên ta chỉ ra m0 lẻ. Thật vậy, nếu m0 chẳn, thì x1 , x2 , x3 , x4 hoặc cùng chẵn hoặc cùng lẻ hoặc có hai chẳn và hai lẻ, chẳng hạn x1 , x2 chẳn và x3 , x4 lẻ. Lúc đó 1 x1 + x2 m0 p = 2 2 2 + x1 − x2 2 2 + x3 + x4 2 2 + x3 − x4 2 2 . Điều này mâu thuẫn với tính nhỏ nhất của m0 . Do đó m ≥ 3, và chúng ta có thể viết x i = bi m 0 + y i , (i = 1, 2, 3, 4), (8) ở đây bi có thể được chọn sao cho | yi |< 21 m0 . Từ (8) và do x1 , x2 , x3 , x4 không đồng thời chia hết m0 , nên ta có 1 0 < y12 + y22 + y32 + y42 < 4( m0 )2 = m20 . 2 Từ (7) và (8) ta thu được y12 + y22 + y32 + y42 ≡ 0(mod m0 ). Vậy tồn tại các số nguyên xi , yi , (i = 1, 2, 3, 4), sao cho x21 + x22 + x23 + x24 = m0 p, y12 + y22 + y32 + y42 = m1 m0 , 0 < m0 < p, 0 < m1 < m0 . Từ đồng nhất thức (6) tồn tại các số nguyên z1 , z2 , z3 , z4 , sao cho z12 + z22 + z32 + z42 = m20 m1 p. 8 (9) Hơn nữa, 4 z1 = 4 x2i (mod m0 ) ≡ 0(mod m0 ). xi (xi − bi m0 ) ≡ xi y i = i=1 4 i=1 i=1 Tương tự z2 ≡ z3 ≡ z4 ≡ 0(mod m0 ). Do đó zi = m0 ti , ti ∈ Z (i = 1, 2, 3, 4). Thay vào (9) ta thu được m1 p = t21 + t22 + t23 + t24 , 0 < m1 < m0 < p. Điều này mâu thuẫn với tính nhỏ nhất của m0 . Vậy định lý được chứng minh hoàn toàn. TÀI LIỆU THAM KHẢO [1] K. Chandrasekharan, Introduction to Analytic Number Theory, Springer, 1968. [2] G.A. Jones and J. Mary Jones, Elementary Number Theory, Springer, 1998. [3] W. Sierpinski and A. Schinzel, Elementary Theory of Numbers, North Holland, 1988. [4] N.C. Ankeny, Sum of three squares, Pro. Amer. Math.Soc. 8 (1957), 316-319. 9 Luật thuận nghịch bậc hai Người trình bày: Văn Đức Trung Nội dung của bài giảng được trình bày theo Chương V của tài liệu [1]. m Trong việc xác định các kí hiệu legendre ( q ) với q là số nguyên tố, ta có p thể quy về việc tính ( q ) với p, q là hai số nguyên tố phân biệt. Nếu p > q p p thì ta phân tích p = kq + p , p < q và ta có ( q ) = ( q ). Trong trường hợp p p < q thì ta sẽ sử dụng luật thuận nghịch Gauss (về mối liên hệ giữa ( q ) và ( pq )) để đưa việc tính ( pq ) về việc tính ( pq ), và khi đó ta lặp lại quá trình p trên. Tuy nhiên cần lưu ý rằng trong quá trình tính ( q ) như trên chúng ta 2 có thể gặp trường hợp tính ( p ), khi đó ta không thể áp dụng luật thuận nghịch Gauss được. Trong chương này chúng tôi sẽ trình bày một phép chứng minh luật thuận nghịch Gauss thông qua việc sử dụng Tổng Gauss và từ đó đưa ra công thức 2 tính ( p ). 1 Luật thuận nghịch Gauss Định lý 1 (Gauss). Cho p, q là hai số nguyên tố lẻ phân biệt, khi đó (q−1) . 2 2 ( pq ) = (−1) (p−1) ( pq ). Do p−1 . q−1 lẻ nếu và chỉ nếu p ≡ q ≡ 3 (mod 4) nên Định lý 1 có thể 2 2 phát biểu như sau: ( pq ) = − ( pq ), nếu p ≡ q ≡ 3 (mod 4) và ( pq ) = ( pq ), trong trường hợp ngược lại. 10 2 Tổng Gauss tổng quát Cho m và n là hai số nguyên khác không. Khi đó tổng Gauss tổng quát được định nghĩa bởi |n| m 2 +πimk eπi n k g(m, n) = . (1) k=1 Chúng ta có mối liên hệ giữa g(m, n) và g(−n, m), xem như là luật thuận nghịch của tổng Gauss, như sau: Định lý 2. Cho m và n là hai số nguyên khác không, khi đó 1 |n| trong đó sgn(r) = 1 πi g(m, n) = e 4 (1−|mn|) sgn(mn) r |r| |m| g(−n, m). (2) nếu r = 0, và sgn(r) = 0 nếu r = 0. Chứng minh. Xét hàm số 2 eπiτ u +2πiXu , Φ(u) = Φ(u, X, τ ) = 2πiu e −1 (3) với u là biến số phức, các tham số X ∈ C và τ ∈ C : Re τ > 0, và tích phân Φ(u)du với C là đường thẳng trong mặt phẳng phức đi qua điểm u = 12 và C hợp với trục số thực một góc là π4 . Φ(u)du là hội tụ. Ta xét hàm Φ Trước hết ta cần chứng minh tích phân C trong một dải bị chặn bởi hai đường thẳng song song với C và nằm về hai phía của C. Khi đó ta có thể biểu diễn iπ u = c + re 4 , τ = Re τ + i Im τ, trong đó c và r là các số thực, c bị chặn và r biến thiên tùy ý. Ta có |eπiτ u và 2 +2πiXu | = e−π Im(τ u πi 2 +2Xu) , τ u2 + 2Xu = iτ r2 + 2e 4 (τ c + X)r + (τ c + 2X)c, 11 suy ra Im(τ u2 + 2Xu) ≥ Re τ.r2 − 2|τ c + X||r| − |(τ c + 2X)c|. Do đó |eπiτ u 2 +2πiXu | ≤ e−πr 2 Re τ +π|τ |(c2 +2|cr|)+2π|X|(|c|+|r|) ≤ Ae−πr 2 Re τ +B|r| , (4) trong đó A và B là các hằng số phụ thuộc r. Hơn nữa √ |e2πiu − 1| ≥ |1 − |e2πiu || = |1 − e− 2πr |. Do r → ±∞ khi |u| → ∞ trong dải đã chọn, nên với |u| đủ lớn thì ta có 1 > 0. 2 |e2πiu − 1| ≥ (5) Từ (4) và (5) ta có |Φ(u)| ≤ A1 e−πr 2 Re τ +B|r| , (6) với |u| đủ lớn trong dải đã chọn. Do đó tích phân Φ(u)du hội tụ. C Với mỗi X ∈ C ta đặt f (X) = f (X, τ ) = Φ(u, X, τ )du. C Với mỗi n > 0 xét hình bình hành γ sinh bởi đường thẳng C, đường thẳng Cn song song với C và cắt trục số thực tại điểm n + 21 , và hai đường thẳng L1 , L2 song song với trục số thực và nằm về hai phía của trục số thực. Theo Định lý Cauchy về thặng dư ta có n eπiτ k Φ(u)du = 2 +2πiXk . (7) k=1 γ Theo (6) thì Φ(u) → 0 đều khi |u| → ∞ trong dải được giới hạn bởi C và Cn . Do đó khi cho L1 và L2 tiến ra vô cùng về hai phía ta có Φ(u)du − Φ(u)du = γ Cn Φ(u)du. C Tuy nhiên, theo (3) ta có Φ(u + n, X) = eπiτ n 2 +2πiXn 12 Φ(u, X + τ n). (8) Do đó Φ(u+n, X)du = eπiτ n Φ(u)du = Cn 2 +2πiXn C Φ(u, X+τ n)du = eπiτ n 2 +2πiXn f (X+τ n). C Kết hợp với (7) và (8) ta có n πiτ n2 +2πiXn e eπiτ k f (X + τ n) − f (X) = 2 +2πiXk . (9) k=1 Mặt khác ta có 2 eπiτ u (e2πi(X+1)u − e2πiXu )du e2πiu − 1 f (X + 1) − f (X) = C eπiτ u = 2 +2πiXu du = e−πi C = e−πi X2 τ X 2 eπiτ (u+ τ ) du C X2 τ 2 eπiτ u du, C trong đó C là ảnh của C qua phép tịnh tiến u → u + Xτ . Bằng cách tính tích phân trong hình bình hành như trên ta có 2 2 eπiτ u du = eπiτ u du, C0 C 13 trong đó C0 là đường thẳng song song với C và đi qua gốc tọa độ. Trên C0 πi ta có u = re 4 , r ∈ R, suy ra +∞ πiτ u2 e du = e πi 4 πi 2 e−πτ r dr = e 4 Iτ , −∞ C0 trong đó +∞ 2 e−πτ r dr. Iτ = (10) −∞ Do đó ta có 1 f (X + 1) − f (X) = eπi( 4 − X2 ) τ Iτ . Thay X bởi X + 1, X + 2, ..., X + m rồi lấy tổng hai vế ta được m−1 1 eπi( 4 − f (X + m) − f (X) = (X+l)2 ) τ Iτ . l=0 Thay X bởi X + τ n − m ta có m−1 1 eπi( 4 − f (X + τ n) − f (X + τ n − m) = (X+τ n−m+l)2 ) τ Iτ . (11) l=0 Từ (9) và (11) suy ra n e πiτ n2 +2πiXn m πiτ k2 +2πiXk f (X+τ n−m)−f (X) = e l=1 n m e πiτ k2 +2πiXk Chọn X = và τ = m , n 1 eπi( 4 − − Iτ k=1 m , 2 l=1 m > 0, n > 0 ta có n m πi m k2 +πimk n e = Im/n e eπi( 4 − −e k=1 = 1 πiτ n2 +2πiXn πi (1−mn) 4 k=1 eπikn+k k=1 14 2 πi n m . (X−k)2 ) τ . (X+τ n−l)2 ) τ Iτ +∞ 2 e−πr dr = 1. Trong công thức trên ta thay m = n = 1 vào ta có I1 = −∞ √ Bằng phép đổi biến r → r τ , trong đó τ là một số thực dương, ta có +∞ 1 2 e−πτ r dr = √ . τ Iτ = −∞ Do đó ta có n πi m k2 +πimk n e k=1 √ m n n πi (1−mn) 2 4 =√ e e−πink−k πi m . m k=1 Suy ra 1 πi 1 √ g(m, n) = √ e 4 (1−mn) g(−n, m). n m (12) Như vậy Định lý được chứng minh với m > 0, n > 0. Nếu m > 0 và n < 0 thì −n, m > 0 nên áp dụng (12) ta có πi 1 1 √ g(−n, m) = √ e 4 (1+mn) g(−m, −n). m −n Suy ra 1 πi 1 g(−m, −n) = √ e− 4 (1−|mn|) g(−n, m). m |n| Mà theo (1) thì g(−m, −n) = g(m, n) nên ta có (2). Nếu m < 0 và n < 0 thì −n, −m > 0 nên áp dụng (12) và g(n, −m) = g(−n, m), g(−m, −n) = g(m, n) ta có (2). 3 Chứng minh luật thuận nghịch Trước hết chúng ta có một mối liên hệ giữa kí hiệu Legendre Gauss g(m, n) như sau: (m n ) và tổng Định lý 3. Cho n là số nguyên tố lẻ và m là số nguyên sao cho (m, n) = 1, khi đó m 1 πi = √ e 4 (n−1) g(m, n). n n 15 (13) Chứng minh. Do k 2 ≡ k (mod 2) nên ta có n n−1 (n+1)k2 πi m n g(m, n) = e m 2 eπi n (n+1)k . =1+ k=1 k=1 Để ý rằng, nếu k 2 ≡ ρ (mod n) thì m m 2 eπi n (n+1)k = eπi n (n+1)ρ . Nhưng với mọi k chạy từ 1 đến n, nếu k 2 ≡ ρ (mod n) thì ρ là thặng dư bậc hai theo modulo n và (n − k)2 ≡ k 2 ≡ ρ (mod n). Do đó khi k chạy từ 1 đến n thì k 2 (lấy theo modulo n) chạy khắp tập các thặng dư bậc hai theo modulo n đúng 2 lần. Vậy m eπi n (n+1)ρ , g(m, n) = 1 + 2 (14) ρ trong đó ρ chạy khắp tập các thặng dư bậc hai theo modulo n. Bây giờ ta xét tổng m eπi n (n+1)ν , ν trong đó ν chạy khắp tập các bất thặng dư bậc hai theo modulo n. Rõ ràng ta có n−1 (n+1)ρ πi m n 1+ e πi m (n+1)ν n e + ρ m eπi n (n+1)k . = ν k=0 m Đặt η = eπi n (n+1) ta có η n = 1 (do n + 1 chẵn) và η = 1 (do n m). Thay vào trên ta có n−1 πi m (n+1)ρ n 1+ e πi m (n+1)ν n + e ρ ηk = = ν k=0 1 − ηn = 0. 1−η (15) Từ (14) và (15) suy ra m m eπi n (n+1)ρ − g(m, n) = ρ eπi n (n+1)ν . (16) ν Ta có 2 trường hợp, m Trường hợp 1: ( n ) = 1. Theo Hệ quả của Định lý 3, Chương IV thì khi ρ chạy khắp tập các thặng dư bậc hai theo modulo n thì ρm cũng vậy. Tương 16 tự, khi ν chạy khắp tập các bất thặng dư bậc hai thì νm cũng vậy. Do đó ta có 1 1 g(m, n) = eπi n (n+1)ρ − eπi n (n+1)ν ρ ν = g(1, n) (theo (16)) = Trường hợp 2: (m n )g(1, n). (m n ) = −1. Tương tự trường hợp 1 ta cũng có 1 1 eπi n (n+1)ν − g(m, n) = ν eπi n (n+1)ρ ρ = −g(1, n) (theo (16)) = (m n )g(1, n). g(m, n) = (m )g(1, n). n Vậy ta luôn có (17) Theo Định lý 2 ta có πi πi 1 √ g(1, n) = e 4 (1−n) g(−n, 1) = e 4 (1−n) n ⇒ g(1, n) = √ (do g(−n, 1) = 1) πi n e 4 (1−n) . Thay vào (17) ta có √1 (m n ) = ne πi (n−1) 4 g(m, n). Bây giờ ta sẽ chứng minh Định lý 1 (luật thuận nghịch Gauss) Trong Định lý 3 thay m = −1 ta có √1 ( −1 n ) = ne πi (n−1) 4 g(−1, n). Mà theo Định lý 2 thì πi πi 1 √ g(−1, n) = e 4 (n−1) g(−n, −1) = e 4 (n−1) , n 17 (do g(−n, −1) = 1) nên ta có ( −1 )=e n πi (n−1) 2 = (−1) n−1 2 . (với n là số nguyên tố lẻ) (18) Theo Định lý 3 và Định lý 2 ta có ( pq ) = √1p e πi (q−1) 4 πi πi 1 g(p, q) = e 4 (q−1) .e 4 (1−pq) √p g(−q, p) πi πi πi −q = e 4 (q−1) e 4 (1−pq) e− 4 (p−1) ( p ). Mà theo (18) thì −1 q ( −q p ) = ( p )( p ) = e πi (p−1) 2 ( pq ), suy ra ( pq ) = e πi (q−1+1−pq+p−1) 4 ( pq ) = e− πi (p−1)(q−1) 4 ( pq ) = (−1) p−1 q−1 . 2 2 ( pq ). (đ.p.c.m) 4 Một số áp dụng 2 Chúng ta có công thức tính ( p ) như sau: Định lý 4. Cho p là một số nguyên tố lẻ, khi đó ta có p2 −1 2 = (−1) 8 . p (19) Hay nói cách khác, 2 = p +1, −1, nếu p = ±1(mod 8), nếu p = ±3(mod 8). Chứng minh. Theo Định lý 3 ta có ( p2 ) = √1p e 18 πi (p−1) 4 g(2, p), và theo Định lý 2 thì πi 1 1 √ g(2, p) = e 4 (1−2p) √ g(−p, 2), p 2 mà theo định nghĩa của g(m, n) ta có g(−p, 2) = 1 + e πip 2 . Như vậy πip 2 2 e−√ 4 ( p ) = 2 (1 + e πip2 ) = √12 (e− πip4 + e πip4 ) = (−1) p 8−1 . 12703 Bây giờ ta sẽ sử dụng các kết quả trên để tính giá trị của ( 16361 ). Do 12703 và 16361 là các số nguyên tố lẻ phân biệt nên áp dụng luật thuận nghịch Gauss ta có 16361 ) = ( ( 12703 16361 12703 ), do 16361 ≡ 3658 (mod 12703) nên ta có Do m n ( mn p ) = ( p )( p ) nên 3658 ( 16361 12703 ) = ( 12703 ). 3658 2 31 59 ( 12703 ) = ( 12703 )( 12703 )( 12703 ) 31 59 = ( 12703 )( 12703 ) (theo Định lý 4) 12703 12703 = −( 31 ).−( 59 ) (theo Định lý 1) 24 18 23 3 2 32 = ( 31 )( 59 )= ( 31 )( 31 )( 59 ) ( 59 ). 22 2 2 32 Do ( 31 )= ( 31 )( 31 ) = 1, và tương tự ( 59 )= 1, nên cuối cùng ta có 19 2 3 2 3 31 1 ( 12703 16361 )= ( 31 )( 31 )( 59 ) = 1.( 31 ).(-1) = ( 3 ) = ( 3 ) = 1. Tài liệu tham khảo [1]. K.Chandrasekharan, Introduction to Analytic Number Theory, Springer, 1968. 20 Một chứng minh cho luật thuận nghịch bậc hai Người trình bày: Nguyễn Thọ Tùng Luật thặng nghịch bậc hai là một trong những kết quả vô cùng đẹp đẽ của số học. Có nhiều cách chứng minh khác nhau cho định lý này. Mục đích của bài viết nhỏ này là đưa ra một chứng minh cho luật thặng nghịch bậc hai dựa vào tổng Gauss. Chứng minh này chủ yếu dựa vào [1]. Luật thuật nghịch bậc hai được phát biểu như sau. Định lí 1. Cho p, q là các số nguyên tố lẻ. Khi đó ta có p q = (−1) p−1 q−1 2 2 q p . Ngoài ra nếu p là một số nguyên tố lẻ thì 2 p = (−1) p2 −1 8 . Chứng minh. Ta trước hết chứng minh cho trường hợp p và q là các số nguyên tố lẻ. Gọi ω là một nghiệm của đa thức X p − 1 trong Fq . Khi đó với mỗi x ∈ Fp ta có thể thấy ω x xác định và không phụ thuộc vào cách chọn đại diện của x trong Fp . Ta xét tổng Gauss sau đây y= x∈Fp x p ωx. Khi đó ta có các khẳng định sau đây Bổ đề 1. y 2 = (−1) 21 p−1 2 p. Thật vậy ta có   x p y2 =  x∈Fp  x p ωx  y∈Fp ωy  . Nhóm các số hạng một cách thích hợp ta được y2 = x,y∈Fp x p y p ω x+y = x,k∈Fp x(k − x) p ωk . Để rút gọn tổng này, ta có thể dùng bổ đề sau. Bổ đề 2. Cho a, b, c là các số nguyên và p là một số nguyên tố sao cho p a và p b2 − 4ac. Khi đó ta có x∈Fp ax2 + bx + c p Bằng cách nhân cả hai về với 4a p =− a p . ta có thể giả sử a = 1 và b = 0. Khi đó đẳng thức của ta được viết lại x2 + c p x∈Fp = −1, với chú ý rằng dấu bằng thực ra xảy ra trong Fp . Đặt h = Euler, ta có h x2 + c h 2l h−l 2 h = (x + c) = x c . p l l=0 p−1 . 2 Theo định lí Cộng tất cả lại ta có x∈Fp x +c p  h 2 = l=0  h h−l  c x2l  . l x∈F p Ở đây ta chú ý rằng xk = x∈Fp p − 1, nếu p − 1|k 0, nếu p − 1 k. 22 (1) Thực vậy, nhóm F∗p là một nhóm cylic. Gọi g là một phần tử sinh của nhóm đó. Khi đó mọi phần tử khác 0 trong Fp đều có dạng g h với h ∈ 0, . . . , p − 2. Do vậy ta có đẳng thức p−1 g ki . k x = i=0 x∈Fp Nếu p − 1|k thì tổng trên hiển nhiên bằng p − 1. Ta xét trường hợp khi p − 1 k; khi đó g k = 1 nên ta có p−2 g (p−1)k − 1 = 0. gk − 1 g ki = i=0 Do vậy chú ý trên được chứng minh. Theo chú ý trên, với l < h thì 2h < p−1 nên tổng x2l = 0. x∈Fp Từ đó ta suy ra được x∈Fp x2 + c p = p − 1 = −1(mod p), (với chú ý là c = 0 nên ch−h = 1). Ta nhận xét rằng vế phải bị chặn bởi p do từng số hạng bị chặn bởi 1. Ngoài ra nếu tồn tại x sao cho p|x2 + c thì có đúng hai số x như vậy do p c. Do đó tổng vế phải luôn chỉ có 0 hoặc 2 số hạng bằng 0, các số hạng khác bằng 1 hoặc −1. Do có đúng p số hạng, tổng này phải là số lẻ. Từ những nhận xét trên, ta suy ra tổng này phải bằng −1. Bổ đề do đó được chứng minh xong. Quay lại bài toán ở trên, khi k = 0 ta có x(k − x) p =− −1 p . Khi k = 0 ta nhận được x∈Fp −x2 p = (p − 1) −1 p . Từ những điều trên ta suy ra được y2 = −1 p ω k + (p − 1) k∈Fp ∗ 23 −1 p = (−1) p−1 2 p. Bổ đề 3. q p y q−1 = . Thật vậy, ta có q  x p yq =  x∈Fp ωx . (2) Do trường Fq có đặc số q nên với mọi a, b ∈ Fq ta đều có (a + b)q = aq + bq . Áp dụng đằng thức này vào (2) ta được x p yq = x∈Fp ω qx . Do gcd(p, q) = 1 nên ánh xạ x → qx là một song ánh từ Fp vào chính nó. Do đó nếu ta đặt t = qx thì tổng trên có thể viết lại yq = q p t p t∈Fp Từ đó ta dễ dàng suy ra được y q−1 = q p = y q−1 = (y 2 ) q−1 2 = ((−1) ωt = q p p−1 2 q p y. . Từ các đẳng thức trên ta có p) q−1 2 = (−1) p−1 q−1 2 2 p q . Đẳng thức này xảy ra trên Fq nhưng do cả hai vế chỉ nhận giá trị 1 hoặc −1 trong trường này nên chúng bằng nhau trên Z. Ta có điều phải chứng minh. Bây giờ ta xét trường hợp 2 p . Chứng minh hoàn toàn tương tự như ở trên. Gọi α là căn bậc 8 của đơn vị trong Fp . Khi đó xét z = α + α−1 Ta có z 2 = 2 do α4 + 1 = 0. Ngoài ra ta cũng có y p = αp + α−p . Nếu p ≡ 1, −1 (mod 8) ta có z p = z. Nói cách khác z p−1 = 1. Ta lại có 2 p = z p−1 = 1. 24 Nếu p ≡ 3, 5 (mod 8) ta có: z p = α5 + α−5 = −(α + α−1 ) = −z. Nói cách khác z p−1 = −1. Tương tự như ở trên ta suy ra 2 p = −1. Tổng hợp lại, ta dễ dàng kết luận được 2 p = (−1) p2 −1 8 . TÀI LIỆU THAM KHẢO [1] J. P. Serre, A course in Arithmetic, Springer, 1996. 25 Một chứng minh sơ cấp cho luật thuận nghịch bậc hai Người trình bày: Hoàng Thị Ngọc Trà Như chúng ta đã biết, luật thuận nghịch bậc hai là một trong những kết quả đẹp của số học. Có nhiều hướng chứng minh khác nhau như hai bài đã đưa trước. Mục đích của bài viết nhỏ này là đưa ra một chứng minh khá sơ cấp cho luật thuận nghịch bậc hai. Chứng minh này chủ yếu dựa theo [1] bằng cách đếm số điểm nguyên. Luật thuận nghịch bậc hai được phát biểu như sau. Định lý 1. Cho p và q là hai số nguyên tố lẻ khác nhau. Khi đó ta có p−1 q−1 q p = (−1) 2 . 2 . q p Để chứng minh định lí này trước hết ta đi chứng minh các bổ đề sau. Bổ đề 1 (Bổ đề Gauss). Cho p là một số nguyên tố lẻ, a ∈ Z, (a, p) = 1. Đặt αj = aj (mod p) với j = 1, . . . , (p−1) , n là số các giá trị của j mà αj > p2 . 2 Khi đó ta có a = (−1)n . p Chứng minh. Gọi r1 , r2 , . . . , rn là các giá trị của αj vượt quá p2 và s1 , s2 , . . . , sk là các giá trị còn lại. Chúng nhận giá trị khác nhau và khác 0. Ta có 0 < p − ri < p/2 với i = 1, . . . , n và không có p − ri = sj . Thật vậy, nếu p − ri = sj thì −ri = sj (mod p), sj = ua(mod p), ri = va(mod p) dẫn tới ua ≡ −va(mod p). Điều này mâu thuẫn với u ≡ −v(mod p). Do vậy ta có thể sắp thứ tự cho dãy s1 , s2 , . . . , p − r1 , . . . , p − rn tương ứng nhận giá trị là 1, . . . , (p−1) . 2 26 Ta có k (p − 1)/2 ! = n (p − ri ) sj j=1 i=1 k (p − 1)/2 ! ≡ (−1) n n sj j=1 ri (mod p) i=1 (p−1)/2) (p − 1)/2 ! ≡ (−1) n αj (mod p) j=1 (p − 1)/2 ! ≡ (−1)n (p − 1)/2 !a(p−1)/2 (mod p). Sử dụng luật giản ước ta được (−1)n ≡ a(p−1)/2 (mod p). Hơn nữa ta có a(p−1)/2 ≡ a (mod p), p (−1)n ≡ a (mod p). p nên ta có Với chú ý rằng hai vế của đồng dư thức này chỉ có thể nhận giá trị 1 hoặc −1, cho nên ta được đẳng thức (−1)n = a (mod p). p Bổ đề 2. Nếu p là một số nguyên tố lẻ sao cho (a, 2p) = 1 thì với (p−1)/2 t= j=1 Đặc biệt với a = 2 ja . p 2 2 = (−1)(p −1)/8 . p 27 a = (−1)t p Chứng minh. Với các hàm được định nghĩa như trên. Xét a ∈ Z, (a, p) = 1. + αj với j = 1, . . . , (p−1) . Chú ý ở đây chúng ta có thể viết ja = p ja p 2 Ta có (p−1)/2 (p−1)/2 ja = j=1 j=1 ja p + p n k rj + j=1 sj . (1) j=1 Hơn nữa, ở phần trên chúng ta đã chứng minh được dãy s1 , . . . , sk , p − r1 , . . . , p − rn sắp xếp được và (p−1)/2 n n (p − rj ) + j= j=1 k j=1 sj = np − j=1 k rj + j=1 sj . (2) j=1 Từ (1) và (2) ta có p−1)/2 (a − 1) (p−1)/2 j=p j=1 j=1 n ja −n +2 rj , p j=1 với chú ý là (p−1)/2 j= j=1 (p2 − 1) 8 nên (p2 − 1) (a − 1) =p 8 (p2 − 1) (a − 1) ≡p 8 (p−1)/2 j=1 (p−1)/2 j=1 n ja −n +2 rj , p j=1 ja − n (mod 2). p Trước hết chúng ta chứng minh cho trường hợp a là số lẻ, tức a − 1 là số chẵn. Suy ra (p−1)/2 n≡ j=1 ja (mod 2). p Theo bổ đề Gauss ta có t = n. Với trường hợp a = 2 thì 2jp = 0 với j = 1, . . . , (p−1) . 2 Suy ra (p2 − 1) n≡ (mod 2). 8 28 Bây giờ, chúng ta sử dụng hai bổ đề trên để chứng minh luật thuận nghịch bậc hai. Chứng minh. p q (p−1) (q−1) q = (−1) 2 . 2 . p Vì p − 1, q − 1 là các số chẵn nên theo bổ đề 2 ta có và p = (−1) q (q−1)/2 lp [q] l=1 q = (−1) p (p−1)/2 kq [p] l=1 , . Vậy để chứng minh mệnh đề trên ta sẽ chứng minh đẳng thức p−1 q−1 . = 2 2 (q−1)/2 l=1 lp + q (p−1)/2 l=1 kq . p Xét tập hợp {kq − lp | k = 1, . . . , q−1 p−1 ; l = 1, . . . , } 2 2 gồm p−1 . q−1 số nguyên khác không. Gọi S1 là số các số nguyên dương và S2 2 2 là số các số nguyên âm trong tập hợp đó, như vậy S1 + S2 = p−1 q−1 . . 2 2 Mặt khác, S1 là số các cặp số nguyên (k, l) thỏa mãn các điều kiện 1 ≤ k ≤ p−1 , 1 ≤ l ≤ q−1 và kq − lp > 0. Với mỗi k cho trước thỏa 1 ≤ k ≤ p−1 thì l 2 2 2 q−1 phải thỏa mãn điều kiện 1 ≤ l ≤ . Vì vậy với mỗi k cho trước có kq p 2 cặp (k, l) thỏa mãn điều kiện nói trên. Từ đó định lí được chứng minh. TÀI LIỆU THAM KHẢO [1] Victor Shoup, A Computational Introduction to Number Theory and Algebra, New York 2008. 29 Hàm số học và điểm nguyên Người trình bày: Trần Thị Hương Giang Bài này đưa ra các ước lượng của các hàm số học. Tài liệu tham khảo là chương VI [1]. 1 Một Số Khái Niệm Định nghĩa 1 (Hàm tổng). Nếu f là một hàm số học thì hàm tổng của f được định nghĩa là N f (n). F (N ) = n=1 Cho f và g là hai hàm xác định trên một tập số thực nào đó. • Big O: Ta nói f (x) = O(g(x)) khi x → ∞ khi và chỉ khi tồn tại hằng số dương M và số thực x0 sao cho |f (x)| ≤ M |g(x)| với mọi x > x0 . • Small o: Ta nói f (x) ∈ o(g(x)) khi x → ∞ khi và chỉ khi với mọi > 0 tồn tại hằng số dương M sao cho |f (n)| ≤ |g(x)| với mọi n ≥ N. Nếu g(x) = 0 thì điều kiện trên tương đương f (x) = 0. x→∞ g(x) lim 2 Bậc của các hàm số học 2.1 Hàm số dàn điểm r(n) 1. Bậc của hàm r(n) Định lý 1. lim inf r(n) = 0. n→∞ 30 Chứng minh. Ta có lim inf r(n) = lim inf r(n) ≤ lim n→∞ N →∞ n≥N inf N →∞ 4k+3≥N r(4k + 3) = 0, vì r(4k + 3) = 0 với mọi k ∈ Z. Mặt khác, vì r(n) ≥ 0 với mọi số nguyên dương n, ta có lim inf n→∞ r(n) ≥ 0. Như vậy chứng tỏ lim inf n→∞ r(n) = 0. Định lý 2. r(n) = O(n ) với mọi > 0. 2. Bậc của hàm tổng R(n) Định lý 3 (Gauss). √ R(N ) = πN + O( N ). Chứng minh. Chứng minh bằng cách đánh giá số điểm nguyên trong miền với thể tích của miền. (a) Thiết lập song ánh: Để xác định R(N ), chúng ta cần ước lượng số điểm nguyên nằm trong và trên đường tròn x2 + y 2 = N . Đây là một bài toán đếm, vì vậy một cách tiếp cận tự nhiên là thiết lập một song ánh. Ta chú ý rằng mỗi điểm nguyên ứng với góc tây nam của đúng một hình vuông đơn vị. Theo cách đó ta có thể gán cho mỗi điểm nguyên với một hình vuông đơn vị. Như vậy, số các điểm nguyên nằm trong và trên đường tròn x2 + y 2 = N sẽ bằng diện tích phủ bởi các hình vuông đơn vị tương ứng của chúng. Bài toán đếm điểm nguyên giờ đây trở thành bài toán tính diện tích. Một vài hình vuông nằm ra ngoài hình tròn một chút. Một vài phần của hình tròn lại không bị hình vuông nào phủ cả. √ 2, nên tất cả các hình (b) Chặn trên: Vì đường chéo của mỗi hình vuông có độ dài √ √ vuông đều nằm trong đường tròn x2 + y 2 = ( N + 2)2 . Vậy nên ta có √ R(N ) < π N+ √ 2 2 . (c) √Chặn √ dưới: Mặt khác, tất cả các hình vuông đều phủ đường tròn bán kính N − 2, nên ta có √ R(N ) > π (d) Ta có N− √ 2 2 , N ≥ 2. √ √ π N − 2 2N + 2 < R(N ) < π N + 2 2N + 2 , suy ra, √ √ √ √ 2π 2π R(N ) − πN √ −2 2π − 2π < −2 2π + √ < < 2 2π + √ < 2 2π + 2π. N N N 31 Vì thế ta có √ |R(N ) − πN | √ < (2 2 + 2)π, N nói cách khác 2.2 √ R(N ) − πN = O( N ). Hàm ước d(n) 1. Bậc của hàm d(n) Định lý 4. lim inf d(n) = 2. n→∞ Chứng minh. Ta thấy rằng khi n > 1, d(n) ≥ 2 (“=” khi và chỉ khi n nguyên tố), vì thế nên lim inf n→∞ d(n) ≥ 2. Mặt khác, ta lại có lim inf d(n) = lim inf d(n) ≤ lim N →∞ n≥N n→∞ inf N →∞ p nguyên tố >N d(n) = lim inf 2 = 2. N →∞ p>N Vì thế ta kết luận lim inf d(n) = 2. n→∞ Định lý 5. Với mọi > 0, luôn tồn tại một dãy số nguyên ni thỏa mãn d(ni ) (log ni ) →∞ khi i → ∞. Thế là chúng ta đã thấy với mọi δ > 0, tồn tại các dãy số ni mà có thể khiến d(ni ) áp đảo (log ni )δ . Vậy hàm d(n) phát triển nhanh đến mức nào? Dưới đây chúng ta sẽ thấy là d(n) vẫn phát triển chậm hơn nδ . Định lý 6. d(n) = o(nδ ), với mọi δ > 0. d(n) → 0 nδ khi n → ∞. Từ Định Lý 26 của phần phụ lục, ta biết rằng chỉ cần chứng minh d(pm ) f (pm ) = mδ → 0 khi pm → ∞ (p nguyên tố, m nguyên dương). p Chứng minh. Giả sử δ > 0. Chúng ta muốn chứng minh rằng f (n) = Ta có f (pm ) = m+1 2m 2 log pm 2 log pm ≤ = ≤ → 0 khi pm → ∞. pmδ pmδ pmδ log p pmδ log 2 32 Định lý 7. Cho > 0, tồn tại N ( ) sao cho log n d(n) < 2(1+ ) log log n , với n > N ( ). Mặt khác, tồn tại vô số các số nguyên n sao cho log n d(n) > 2(1− ) log log n . 2. Bậc của hàm tổng D(n) Định lý 8. D(N ) = N log N + O(N ). Chứng minh. Ta có N n=1 1. 1= d(n) = D(N ) = 1≤n≤N xy=n 1≤xy≤N Như vậy, D(N ) là số các điểm nguyên ở góc phần tư thứ nhất của mặt phẳng (x, y) mà thuộc hoặc nằm dưới đường hyperbol xy = N , nhưng không nằm trên trục Ox, Oy. Như vậy các điểm nguyên này thuộc phần mặt phẳng bên trái đường x = N và bên dưới đường y = N trong góc phần tư thứ nhất. Chúng ta sẽ đếm những điểm nguyên nói trên theo cách sau. Từ mỗi điểm 1 ≤ a ≤ N , chúng ta sẽ đếm số điểm nguyên thuộc đường x = a mà nằm giữa hyperbol xy = N và trục Ox, sau đó chúng ta sẽ cộng số điểm nguyên trên tất cả các đoạn x = a lại. Ta N dễ thấy số điểm nguyên như trên thuộc mỗi đoạn x = a là .Vậy ta có a N D(N ) = a=1 33 N . a Đặt θa = N N − a a (suy ra 0 ≤ θa < 1), vậy thì N D(N ) = N a=1 vì N a=1 θa 1 − a N N θa = N a=1 a=1 1 + O(N ), a < N . Từ Hệ Quả 1 của Định Lý 27 (Phụ Lục 2), chúng ta suy ra D(N ) = N log N + O(N ). Định lý 9 (Dirichlet). √ D(N ) = N log N + (2γ − 1)N + O( N ), với γ là hằng số Euler. Chứng minh. Bây giờ chúng ta sẽ đếm số điểm nguyên tương ứng với D(N ) theo cách khác. Ta nhận thấy số điểm nguyên nói trên bằng hai lần số điểm nguyên trong ABGEO trừ đi số điểm nguyên trong OF GE. Vì thế ta có √ D(N ) = 2 1 − [ N ]2 √ 1≤x≤ N 1≤xy≤N √ 1 − [ N ]2 =2 √ 1≤x≤ N 1≤y≤N/x =2 1≤x≤N 34 √ N − [ N ]2 . x Đặt θx = N/x − [N/x] và θ = √ √ N − [ N ] (suy ra 0 ≤ θx , θ < 1). Ta có D(N ) = 2N √ 1≤x≤ N = 2N √ 1≤x≤ N = 2N √ 1≤x≤ N vì 1 −2 x √ 1≤x≤ N 1 −N −2 x √ θx − ( N − θ)2 √ 1≤x≤ N √ θx + 2θ N − θ2 . √ 1 − N + O( N ). x √ θx = O( N ), θ2 = O(1). √ 1≤x≤ N Từ Hệ quả 1 của Định lý 27 (Phụ Lục 2) ta suy ra định lý được chứng minh. √ G.Voronoi đã cải tiến sai số O( N ) thành O(N 1/3 log N ). Người ta phỏng đoán rằng 1 sai số chính xác là O(N 4 + ) với > 0 bất kỳ. Mặt khác, chúng ta biết rằng sai số không phải là O(N 1/4 ). 2.3 Hàm Euler ϕ(n) 1. Hàm ϕ(n) Chúng ta nhắc lại một số điều liên quan đến hàm Euler ϕ(n). Ta biết rằng: • Nếu n > 1 thì ϕ(n) < n, • Nếu n = pm , với p nguyên tố, m ≥ 1 và p > 1/ > 1 thì ϕ(n) = n 1 − 1 p > n(1 − ). Từ hai bất đẳng thức trên ta có kết quả sau. Định lý 10. lim sup n→∞ ϕ(n) = 1. n Một định lý khác có liên quan đến bậc của ϕ(n) là Định lý 11. Với mọi δ > 0, ta có ϕ(n) → ∞, khi n → ∞. n1−δ Chứng minh. Nếu δ > 1 thì điều cần phải chứng minh là hiển nhiên. Nếu δ ≤ 1, ta đặt n1−δ f (n) = . Thế thì f sẽ có tính nhân, và theo Định Lý 26, ta chỉ cần chứng minh ϕ(n) f (pm ) → 0 khi pm → ∞. 35 Trên thực tế, với mọi δ > 0, ta có ϕ(pm ) 1 1 = = pmδ 1 − m m(1−δ) f (p ) p p 1 ≥ pmδ → ∞ khi pm → ∞. 2 Từ Định Lý 10 và Định Lý 11, ta thấy rằng giả thuyết ϕ(n) = O(n ) là sai với tất cả < 1. 2. Hàm tổng Φ(n) Định lý 12 (Mertens). Φ(t) = 3t2 + O(t log t). π2 Chứng minh. Chúng ta muốn dùng công thức nghịch đảo Mobius thứ hai (xem phần Ứng Dụng) để viết lại công thức của Φ(t). Tức là ý tưởng ban đầu của ta là viết Φ(t) = 1≤d≤t µ(d)g(d) với g(d) = 1≤d≤t Φ(t). Thực ra nếu dùng trực tiếp hàm Φ như vậy thì công thức của g không được đẹp. Vì vậy thay vì dùng Φ, ta sẽ xét hàm Ψ(t) = 2φ(t) − 1. Chú ý, xét về ý nghĩa hình học, ta có Φ(t) = 1= 1≤n≤t 1≤m≤n (m,n)=1 1. 1≤m≤n≤t (m,n)=1 Vì vậy Φ(t) cho ta có điểm nguyên có tọa độ nguyên tố cùng nhau, nằm trong tam giác vuông 0 < y ≤ x ≤ t. Dễ dàng nhận thấy ψ(t) là số điểm nguyên trong cả hình vuông 0 < x ≤ t, 0 < y ≤ t. Bây giờ, ta có ψ 1≤d≤t t d 1 = [t]2 = = 1≤d≤t 0 2 thì (a − 1) | (an − 1) mà 1 < a − 1 < an − 1. Thực ra, trong trường hợp đó, n sẽ phải là số nguyên tố. Giả sử n = kl, 1 < k < n. Thế thì (2k − 1) | (2n − 1), mà 1 < 2k − 1 < 2n − 1. 4 Phụ lục 4.1 Nhắc lại định nghĩa Định nghĩa 4 (Hàm số học). Hàm số học là hàm số từ tập số nguyên dương đến tập số phức. Đa số các hàm số học mà chúng ta sẽ xem xét trong phần này nhận các giá trị nguyên. Định nghĩa 5 (Tính nhân). Một hàm số học f được gọi là có tính nhân nếu nó thỏa mán • f không phải là hàm 0 (hoặc có thể thay bằng điều kiện f (1) = 1), • f (mn) = f (m)f (n), nếu (m, n) = 1. Ví dụ 2. Hàm Euler ϕ (đã giới thiệu ở chương II) là một hàm số học có tính nhân. Định nghĩa 6 (Hàm điểm nguyên trên đường tròn r(n)). Với mọi số nguyên dương n, hàm số học r(n) cho ta số các cách biểu diễn n dưới dạng tổng của hai bình phương của số nguyên. Nói cách khác, r(n) = #{(x, y) ∈ Z × Z | x2 + y 2 = n}. Định nghĩa 7. Hàm ước d : N → N được định nghĩa như sau. Với mọi số nguyên dương n, d(n) cho số các ước nguyên dương của n. 42 Định nghĩa 8 (Hàm von Mangoldt Λ). Hàm von Mangoldt Λ là hàm số học định nghĩa như sau log p nếu n là mũ nguyên tố pm , m > 0, Λ(n) = 0, cho những trường hợp còn lại Một hàm có liên quan chặt chẽ tới hàm d(n) là hàm σ(n), hàm số tính tổng các ước dương của n. Tổng quát hỏa hàm σ(n), chúng ta có định nghĩa sau. Định nghĩa 9 (σk (n)). Hàm σk (n) : N → N được định nghĩa bởi dk , , k = 0, 1, 2, . . . σk (n) = d|n Chú ý, σ0 (n) = d(n) và σ1 (n) = σ(n). Định lý 20. Hàm số học σk (n) có tính nhân. Định lý 21. Nếu n = r i=1 pai i > 1 thì ta có r (ai +1)k pi σk (n) = pki i=1 −1 . −1 Chứng minh. Vì σk có tính nhân, chúng ta có σk (pai i ) σk (n) = r r r i=1 = (1 + pki + p2k i + ··· + piai k ) = i=1 i=1 (ai +1)k pi pki −1 . −1 Chú ý nếu k = 1 thì ta có r σ1 (n) = σ(n) = i=1 (ai +1) pi −1 . pi − 1 Định nghĩa 10 (µ(n)). Hàm Mobius là hàm số học được định nghĩa bởi ba tính chất sau • µ(1) = 1 • µ(n) = (−1)k , nếu n là tích của k số nguyên tố khác nhau • µ(n) = 0 nếu n chia hết cho bình phương của bất cứ số nguyên nào khác 1. Từ định nghĩa chúng ta dễ dàng nhận thấy: Định lý 22. Hàm Mobius µ có tính nhân. 43 Định lý 23. Nếu f là một hàm số học, và nếu g(n) = f (d), d|n thì ta có f (n) = µ(d)g d|n n . d Định lý 24. Ta có 1, 0, µ(d) = d|n nếu n = 1, if n > 1. Chứng minh. Trường hợp n = 1 là hiển nhiên. Giả sử n = dương d của n mà thỏa mãn µ(d) = 0 là m i=1 pai i > 1. Thế thì các ước 1, p1 , p2 , . . . , pm , pi pj (i = j), pi pj pk (i = j = k = i), . . . , p1 p2 . . . pm . Nên ta có µ(d) = µ(1) + d|n vì thế µ(d) = 1 − d|n µ(pi pj ) + · · · + µ(p1 p2 . . . pm ), µ(pi ) + i i 0 với t ≥ 1, thì X g(n) = 1≤n≤X g(t)dt + A + O(g(X)), 1 với n nguyên dương, X ≥ 1, và A là hằng số chỉ phụ thuộc vào g. 44 Hệ quả 1. Tồn tại hằng số γ (hằng số Euler) thỏa mãn 1≤n≤X 1 = log X + γ + O n 1 X . Hệ quả 2. Vì X 2 ta có 2≤n≤X dt = log log X − log log 2, t log t 1 = log log X + B + O n log n 1 X log X , với B là một hằng số. TÀI LIỆU THAM KHẢO [1] K. Chandrasekharan, Introduction to Analytic Number Theory, Springer, 1968. 45 46 µ(1) = 1 µ(n) = (−1)r nếu ai ≤ 1 µ(n) = 0 các t.h khác #{1 ≤ k ≤ n | (k, n) = 1} µ(n) ϕ(n) dk σk (n) = σ(n) d|n Hàm Định nghĩa r(n) #{(x, y) ∈ Z2 | x2 + y 2 = n} d(n) #{d | d|n} + 1) pi i r i=1 pki −1 (a +1)k−1 r i=1 (ai n p|n 1− 1 p µd|n µ(d) = 0 nếu n > 1 σk (n) = d(n) = Công thức φ(n) n1−δ →∞ Bậc O(n ) d(n) = o(nδ ) Φ(N ) Hàm tổng R(N ) D(N ) Φ(t) = 3t2 π2 + O(t log t) Bậc Trung Bình√ R(N ) = πN + O( N ) D(N ) = N log N + O(N ) √ D(N ) = N log N + (2γ − 1)N + O( N ) Bảng Tổng Kết Các Hàm Số Học Thường Gặp Định lý Chebyshev về sự phân bố số nguyên tố Người trình bày: Phạm Ngọc Hoàng Minh Nội dung bài này trình bày dựa theo chương VII [1], cái đưa ra những đánh giá trung gian của định lý về số nguyên tố thông qua hàm Chebyshev. 1 Các hàm Chebyshev Như ta đã biết, nếu gọi π(x) là số các số nguyên tố bé hơn hay bằng x thì π(x) −→ ∞ khi x −→ ∞. Trong chương này, ta sẽ chứng minh một số kết π(x) −→ 1 quả trung gian trước khi chứng minh, trong chương XI, rằng x/ log(x) khi x −→ ∞. 1 và tích p trên toàn bộ tập số nguyên tố. Định lý 1 (Euler). Tổng ( 1 ) đều phân kỳ, khi p chạy 1 − 1/p Chứng minh. Trước hết, ta chứng minh tích đã nêu là phân kỳ. Với k nguyên dương, ta đặt 1 P (k) = . 1 − 1/p p≤k Khi đó, lấy m nguyên dương, sao cho k < 2m , ta có 1 1 P (k) ≥ (1 + 2 + · · · + m ) ≥ p p p≤k 1 phân kỳ, nên n nguyên tố. Ta lại có Mà k n=1 1 . n 1 phân kỳ khi p chạy qua toàn bộ tập số 1 − 1/p 1 1 1 ≥ ≥ log( ), pn pn−1 − 1 1 − 1/pn−1 47 với p1 < p2 < · · · < pk < · · · là dãy tất cả các số nguyên tố. Vì thế, log( 1 )≤ 1−p 1 . p 1 1 phân kỳ, dẫn đến log( ) phân kỳ, nên 1 − 1/p 1−p khi p chạy qua toàn bộ tập số nguyên tố. 1 phân kỳ p Mà Định nghĩa 1. Các hàm ϑ và ψ Các hàm Chebyshev ϑ và ψ được định nghĩa như sau ϑ(x) = log(p), p≤x với x > 0 và p là số nguyên tố; và ψ(x) = log(p), pm ≤x với tổng chạy qua mọi cặp p, m với p nguyên tố và m nguyên dương. Các công thức khai triển của ψ 1. Ta đã biết hàm Von Mangoldt định nghĩa bởi Λ(n) = log(p) khi n = pm và Λ(n) = 0 tại các trường hợp khác. Khi đó ψ(x) = Λ(n). n≤x 2. Ta có eϑ(x) là tích của tất cả các số nguyên tố p x, eψ(x) là bội chung nhỏ nhất của các số nguyên nhỏ hơn hay bằng x. Nếu pm x thì 1 m p x nên ta có ∞ 1 ϑ(x k ). ψ(x) = k=1 Chú ý tổng trên hữu hạn do ϑ (z) = 0, ∀z < 2. 3. Nếu pm x < pm+1 thì ln(p) được lặp lại m lần và m = đó, ta có ψ(x) = p x log(x) log p. log(p) Bây giờ ta thiết lập mối quan hệ của 48 π(x) , ϑ(x) , x/ log x x ψ(x) x . log(x) log(p) . Do Định lý 2. Đặt π(x) log(x) ; x→∞ x π(x) log(x) = lim sup ; x x→∞ ϑ(x) = lim inf ; x→∞ x ϑ(x) ; = lim sup x x→∞ ψ(x) = lim inf ; x→∞ x ψ(x) . = lim sup x x→∞ l1 = lim inf L1 l2 L2 l3 L3 Khi đó l1 = l2 = l3 và L1 = L2 = L3 . Chứng minh. Do các dạng khai triển của ψ(x) như đã nói ở trên, ta sẽ có được bất đẳng thức sau ϑ(x) ≤ ψ(x) ≤ π(x) log(x). Vì vậy L2 ≤ L3 ≤ L1 . Ta lại có log(p) ≤ ϑ(x), xα (2n + 1)P 2 > 2nP 2 > 2n 22n 2 √ N , nên N < . Từ (2n)4 2n đây, ta suy ra ngay được log(N ) < 2n log(2) − 1 log(2n). 2 Mà (2n!) (n + 1) . . . (2n) = n!n! n! nên N chia hết cho mọi số nguyên tố n < p ≤ 2n. Vì thế, N= (ϑ(2n) − ϑ(n)) ≤ log(N ). Suy ra (ϑ(2n) − ϑ(n)) < 2n log(n) − 1 log(2n), 2 với mọi n ≥ 2. Tiếp theo, ta chứng minh bất đẳng thức ϑ(k) < 2k log(2) với mọi k ≥ 1. Thật vậy, trước hết, ta kiểm tra đẳng thức cho k = 1, . . . , 14 và sau đó chứng minh quy nạp theo k bằng cách áp dụng (ϑ(2n) − ϑ(n)) < 2n log(n) − 1 log(2n), 2 với mọi n ≥ 2 và chú ý rằng ϑ(2i + 1) ≤ ϑ(2i + 2). Như vậy, ta đã chứng minh được một nửa bài toán và chỉ ra A = 2 log(2) > 0 sao cho ϑ(k) ≤ Ak với mọi k ≥ 2. (2n!) n Bây giờ, ta xét nửa còn lại. Do N = C2n = , nên theo công thức tính n!n! số mũ của 1 số nguyên tố trong bổ đề trên, ta có thể viết pνp , N= p≤n 51 với 2n n −2 r r p p νp = r≥1 . Khi đó, do ta có [2x] − [x] ≤ 1, nên tp νp ≤ 1 = tp , r=1 với tp là số nguyên không âm thỏa mãn ptp là ước của 2n nhưng ptp +1 không log(2n) là ước của 2n, nghĩa là tp = . Do đó, log(p) log N ≤ p≤2n log(2n) log(p) log(p) = ψ(2n). 22n Mà √ < N , nên (2n − 1) log(2) − 12 log n < ψ(2n), với mọi n ≥ 2. Vì thế, 2 n với n đủ lớn ta có 1 log(n) ≤ n log(2). 2 Khi đó, (2n − 1) log(2) − n log(2) ≤ ψ(2n). ψ(2n) . Tương tự như trên, và 2n chú ý (m − 1) log(2) ≤ ψ(2m) ≤ ψ(2m + 1), ta sẽ có được Suy ra (n − 1) log(2) ≤ ψ(2n), nên 1 4 log(2) ≤ 1 ψ(2m + 1) log(2) ≤ . 8 2m + 1 Vậy, ta đã giải quyết nửa còn lại của bài toán và chỉ ra a = ak ≤ ψ(k) với k đủ lớn. 3 log(2) 8 > 0 để Định đề Bertrand Định lý 4 (Chebyshev). Cho n là một số nguyên dương. Khi đó tồn tại một số nguyên tố p sao cho n < p 2n. 52 Chứng minh. Chúng ta sẽ chứng minh ϑ(2n) − ϑ(n) > 0 khi n > 26 và kiểm tra cho các trường hợp còn lại. Xét n = N = C2n (2n)! = pνp , n!n! p≤2n νp = 2n n − 2 pr pr với r≥1 . Phân các số nguyên tố p trong tích trên thành 4 loại và khi đó νp sẽ có giá trị như sau đối với mỗi loại • Loại 1: n < p ≤ 2n. Khi đó, νp = 1. • Loại 2: 32 n < p ≤ n. Khi đó, do p > 2n n lại có < 3 và 1 ≤ , suy ra p p νp = 2 • Loại 3: √ 27 3 > 3 nên p2 > 3p ≥ 2n. Hơn nữa, ta 2n n −2 ≤ 2 − 2.1. p p 2n < p ≤ n. Khi đó, do p2 > 2n nên νp = 2 • Loại 4: p < có √ 2n n −2 ≤ 1. p p 2n. Khi đó, lý luận giống với trong chứng minh định lý 3, ta νp ≤ log(2n) = Mp . log(p) Áp dụng các chặn của νp ở trên, ta suy ra log(N ) ≤ log(p) + n 26 . Vậy, từ bất đẳng thức trên, ta chỉ cần chứng minh √ √ 2n 1 0 < 2n log(2) − log(2) − log(n) − ϑ( ) − ϑ( 2n) + π( 2n) log(2n). 2 3 Khi đó, do các bất đẳng thức đã đề cập trong chứng minh định lý 3 và xét √ √ 2n 4n tại n > 26 , ta có ϑ( ) ≤ log(2), và ϑ( 2n) ≥ π( 2n) log(2), cùng với 3 3 √ √ π( 2n)(− log(2) + log(2n)) ≤ 12 2n log(n). Ta suy ra rằng để chứng minh bất đẳng thức √ √ 1 2n 0 < 2n log(2) − log(2) − log(n) − ϑ( ) − ϑ( 2n) + π( 2n) log(2n), 2 3 ta chỉ cần chứng minh √ 4n 1√ 2n log(n), log(2) − 0 < 2n log(2) − log(2 n) − 3 2 hay có thể viết lại là 0< √ √ √ 3 log(n) 3 2 log( 4n) √ 2n − − . 2 log(2) log(2) 4n Ta có thể chứng minh bất đẳng √ thức cuối√cùng bằng việc khảo sát giá trị các √ 3 2 log( 4x) 3 log(x) √ hàm số 2x − và − . Cuối cùng, để ý rằng mọi số 2 log(2) log(2) 4x nguyên tố trong dãy 2, 3, 5, 7, 13, 23, 43, 67 đều nhỏ hơn 2 lần số nguyên tố ngay sau đó trong dãy. Ta kết thúc chứng minh cho định lý này. 4 Đẳng thức Euler và công thức tổng Abel ∞ Đẳng thức n=1 1 ns = p 1 1−p−s , σ > 1. Khi p chạy qua tập tất cả các số nguyên tố, được coi là bước đầu tiên trong việc gắn kết lý thuyết số và giải tích. Ta xét định lí sau đây. Định lý 5 (Euler identity). Cho f là một hàm số học nhân tính (multiplica∞ f (n) hội tụ tuyệt đối. Khi đó, tive arithmetical function), thỏa mãn dãy n=1 ta có đẳng thức sau ∞ 1 + f (p) + f (p2 ) + · · · , f (n) = n=1 p 54 với tích vô hạn bên tay phải là hội tụ tuyệt đối. Nếu f nhân tính toàn bộ, nghĩa là: f (mn) = f (m)f (n) với mọi m, n nguyên dương thì ∞ 1 f (n) = . 1 − f (p) n=1 p Chứng minh. Trong chứng minh này, ta dùng p, q để chỉ các số nguyên tố. ∞ ∞ f (n) hội tụ tuyệt đối nên zp = Do n=1 f (pr ) và r=1 zp hội tụ tuyệt đối. Đặt p 1 + f (p) + f (p2 ) + · · · = PM = p≤M (1 + zp ), p≤M và QM,m = (1 + f (p) + f (p2 ) + · · · + f (pm )), với mọi M , m nguyên dương. Cho ε > 0. Khi đó, do PM −→ (1 + zp ) và p ∞ f (n) hội tụ tuyệt đối nên tồn tại Mε nguyên dương, sao cho n=1 ε (1 + zp )| < , 2 |PMε − p và ∞ k=Mε ( ) ε |f (k)| < . 4 Với Mε như trên, tồn tại mε > log2 Mε , sao cho ∞ |f (j)| < j=2mε ε , 4π(Mε )Rε với π(Mε ) là số các số nguyên tố không vượt quá Mε và Rε = max | q≤Mε q=p≤M ε (1 + zp )|. Do mε > log2 Mε nên {1, 2, . . . , Mε } ⊂ {pα1 1 . . . pαk k |k = π(Mε ), p1 < p2 < · · · < pk , 0 ≤ αi ≤ mε , ∀0 < i < k + 1} = T, suy ra Mε QMε ,mε = f (j) + j=1 f (i). i∈T \{1,2,...,Mε } 55 Do đó, ∞ ∞ | ε |f (k)| < . 4 |f (n)| − QMε ,mε | < n=1 k=Mε ( ) Ta lại có ∞ mε PM ε = r f (pr ) , f (p ) − 1+ r=1 p≤Mε r=mε +1 suy ra PMε = QMε ,mε + Bq q≤Mε ∞ với Bq = (1 + zp ) , q=p≤Mε f (q r ). Vì thế nên r=mε +1   |QMε ,mε − PMε | ≤ Rε  |Bq | . q≤(Mε ) Mà ∞ ∞ |Bq | ≤ |f (j)| ≤ j=q mε +1 |f (j)| < j=2mε +1 ε , 4π(Mε Rε ) ta suy ra |QMε ,mε − PMε | < ( 1)Rε q≤Mε Từ ( ), ( ), và ( ε ε = . 4π(Mε )Rε 4 ( ) ), ta suy ra ∞ | 1 + f (p) + f (p2 ) + · · · | f (n) − n=1 p ∞ ≤ |PMε − |f (n)|−QMε ,mε |+|QMε ,mε −PMε | < (1 + zp )|+| p n=1 Mà ε > 0 bất ỳ nên ta phải có ∞ 1 + f (p) + f (p2 ) + · · · . f (n) = n=1 p 56 ε ε ε + + = ε. 2 4 4 Định lý 6 (Công thức tổng Abel). Cho 0 < λ1 ≤ λ2 ≤ · · · là một dãy các số thực thỏa mãn λn → ∞ khi n → ∞ và (an )n∈N là một dãy số phức. Đặt A(x) = an . Giả sử ϕ(x) là một hàm nhận giá trị phức, xác định trên λn ≤x [0, ∞). Khi đó ta có k k−1 an ϕ(λn ) = A(λk )ϕ(λk ) − A(λn ) [ϕ(λn+1 ) − ϕ(λn )] . n=1 i=1 Nếu ϕ có đạo hàm liên tục trên (0, ∞) và x ≥ λ1 thì đẳng thức trên có thể được viết thành x an ϕ(λn ) = A(x)ϕ(x) − A(t)ϕ (t)dt. λ1 λn ≤x Hơn nữa, nếu A(x)ϕ(x) → 0 khi x → ∞ thì ∞ ∞ an ϕ(λn ) = − A(t)ϕ (t)dt, λ1 n=1 với điều kiện là một trong hai vế hội tụ. Chứng minh. Ta viết lại k−1 A(λn ) [ϕ(λn+1 ) − ϕ(λn )] n=1 k k−1 A(λn−1 )ϕ(λn ) − = n=2 A(λn )ϕ(λn ) n=1 k−1 = A(λk−1 )ϕ(λk ) + ϕ(λn ) [A(λn−1 ) − A(λn )] − A(λ1 )ϕ(λ1 ) n=2 k−1 = A(λk−1 ϕ(λk ) − A(λn )an n=2 57 − a1 ϕ(λ1 ). Vậy ta có k−1 A(λk )ϕ(λk ) − A(λn ) [ϕ(λn+1 ) − ϕ(λn )] n=1 k−1 = [A(λk ) − A(λk−1 )] ϕ(λk ) + an ϕ(λn ) + a1 ϕ(λ1 ) n=2 k−1 = ak ϕ(λk ) + k an ϕ(λn ) + a1 ϕ(λ1 ) = n=2 ak ϕ(λk ). n=1 Nếu ϕ có đạo hàm liên tục trên (0, ∞) và x ≤ λ1 , ta chọn k sao cho λk ≤ x ≤ λk+1 . Khi đó ta có k−1 an ϕ(λn ) = − λn ≤x A(λn ) [ϕ(λn+1 ) − ϕ(λn )] + A(λk )ϕ(λk ). n=1 Tổng này có thể viết lại dưới dạng λk x A(λk )ϕ(λk ) − A(t)ϕ (t)dt = A(x)ϕ(x) − λ1 (Do A(x) = kn=1 an ϕ(λn )) Nếu A(x)ϕ(x) → 0 khi x → ∞ nên chỉ cần hội tụ thì ta sẽ có cả hai vế hội tụ và ∞ ∞ n=1 an ϕ(λn ) hoặc x λ1 A(t)ϕ (t)dt x an ϕ(λn ) = A(t)ϕ (t)dt. λ1 n=1 5 A(t)ϕ (t)dt. λ1 Hàm Zeta của Riemann Định nghĩa hàm ζ bằng chuỗi hội tụ Cho s là số phức , s = σ +it , với σ > 1, ta định nghĩa hàm Zeta của Reimann như sau ∞ 1 ζ(s) = . s n n=1 Khi đó, do σ > 1 và | 1 ns |= 1 nσ ∞ nên n=1 nghĩa tốt. 58 1 ns hội tụ tuyệt đối và ζ(s) được định Định lý 7. Ta có thể xem hàm ζ như là một hàm phân hình trên nửa mặt phẳng σ > 0. Khi đó, ζ có một cực đơn tại 1 với thặng dư là 1. Chứng minh. Dùng công thức tổng Abel với dãy an = 1 và hàm ϕ(x) = ,x > 1 ta có ∞ ∞ [t] 1 ζ(s) = = s dt, s s+1 n t 1 n=1 1 xs với s = σ + it , σ > 1. Mà ∞ s dt = s ts+1 1 ∞ ⇒s 1 ∞ [t] t 1 t1−s t→∞ dt = s | −s ts+1 1 − s t=1 Xét ϕ(z) = ∞ {t} dt, 1 tz+1 dt − s s+1 ∞ [t] ∞ t 1 1 {t} dt ts+1 {t} 1 dt = 1 + −s ts+1 s−1 ∞ 1 {t} dt. ts+1 Re(z) > 0. Ta có • ϕ được định nghĩa tốt trên nửa mặt phẳng Re(z) > 0. • ϕ liên tục trên nửa mặt phẳng Re(z) > 0. (Dùng định lý hội tụ tuyệt đối của Lebesgue) Bây giờ , ta sẽ chứng minh ϕ có đạo hàm tại mọi điểm trên nửa mặt phẳng Re(z) > 0. Thật vậy, cho z◦ ∈ {Re(z) > 0} và zn ⊂ {Re(z) > 0}, sao cho zn −→ z◦ khi n −→ ∞. Ta sẽ chứng minh ϕ(zn ) − ϕ(z◦ ) −→ zn − z◦ ∞ 1 {−t} log(t)dt, tz+1 khi n −→ ∞. Nghĩa là chứng minh ∞ 1 {t} 1 1 (( )zn +1 − ( )z◦ +1 )dt −→ zn − z◦ t t ∞ 1 {−t} log(t)dt, tz+1 khi n −→ ∞. Đặt fn (t) = {t} 1 1 (( )zn +1 − ( )z◦ +1 ), zn − z◦ t t f◦ (t) = {−t} log(t). tz+1 59 Ta sẽ có được ∀t ∈ (1, ∞), fn (t) −→ f◦ (t), khi n −→ ∞. Viết zn = xn + iyn , z0 = x0 + iy0 . Khi đó zn −→ z0 dẫn đến xn −→ x0 và yn −→ y0 khi n −→ ∞. Vì thế ∃a > 0 sao cho a < x0 , và a < xn , với mọi n nguyên dương. Ta lại có ∀t > 1 thì 1 1 1 | ( )zn +1 − ( )z◦ +1 ) | | xn − x0 | + | y n − y 0 | t t 1 1 1 1 1 | ( )xn +1+iyn − ( )x0 +1+iyn + ( )x0 +1+iyn − ( )x0 +1+iy0 | ≤ | xn − x0 | + | y n − y 0 | t t t t 1 1 xn +1+iyn 1 x0 +1+iyn 1 1 x0 +1+iyn 1 ≤ |( ) −( ) |+ |( ) − ( )x0 +1+iy0 | . | xn − x0 | t t | yn − y0 | t t | fn (t) |≤ Mà 1 1 1 1 1 1 | ( )xn +1+iyn − ( )x0 +1+iyn | = | ( )xn +1 − ( )x0 +1 | | xn − x0 | t t |xn − x0 | t t 1 1 = ( )ξn +1 log(t) ≤ ( )a+1 , t t với mọi t > 1, ở đây, ξn ∈ I(xn , x0 ). Và ta còn có 1 1 1 |( )x0 +1+iyn − ( )x0 +1+iy0 | |yn − y0 | t t 1 x0 +1 1 ( ) | cos(yn log(1/t)) + i sin(yn log(1/t)) − cos(y0 log(1/t)) − i sin(y0 log(1/t))| = |yn − y0 | t 1 = ( )x0 +1 | − log(1/t) sin(βn log(1/t)) + i log(1/t) cos(γn log(1/t))| t 2 log(t) ≤ x0 +1 . t Đặt g(t) = log(t) 2 log()t + x0 +1 . ta+1 t Khi đó, do a > 0, x0 > 0 nên g ∈ L1 ((1, +∞)) và |fn (t)| ≤ g(t) với mọi n nguyên dương và t > 1. Vậy theo định lý hội tự tuyệt đối thì lim là tồn tại và ϕ có đạo hàm tại z0 . Vậy ϕ giải tích trên nửa mặt phẳng Re(z) > 0. 1 + 1 − sϕ(s) nên ζ là hàm phân hình trên nửa mặt phẳng Mà ζ(s) = s−1 Re(s) > 0, với cực đơn tại s = 1 là cực duy nhất, và tại đó thặng dư bằng 1. 60 1 hội tụ đều về ζ(s) trên nửa mặt phẳng 1 − p−s p Re(s) > 1 + δ, với δ > 0 bất kỳ. Từ đó, suy ra Định lý 8. Tích −ζ (s) = ζ(s) ∞ n=1 Λ(n) , n khi Re(s) > 1. Chứng minh. Xét dãy (ap,M ) như sau ap,M = 1/ps nếu p ≤ M , và ap,M = 0 nếu p > M . Khi đó, với mọi n nguyên dương, n > 1, và có phân tích ra thừa số nguyên tố là n = pα1 1 . . . pαk k . Đặt fM (n) = aαp11,M . . . aαpkk,M và fM (1) = 1. Khi ∞ 1 đó, f là hàm nhân tính và fM (n) hội tụ tuyệt đối (do |fM (n)| ≤ Re(s) ). n n=1 ∞ 1 . Vậy, theo đẳng thức Euler, ta có PM = fM (n), với PM = 1 − p−s n=1 p≤M Do đó, 1 1 − p−s | p≤M Suy ra | n=1 n=1 |1/ns |. 1/ns | ≤ fM (n) − − ζ(s)| = | n≥M 1/n1+δ , với mọi s thỏa Re(s) > 1 + δ. Vậy −ζ(s)| ≤ p≤M ∞ ∞ n≥M 1 hội tụ đều về ζ(s) trên nửa mặt phẳng Re(s) > 1 + δ. Mà 1 − p−s hàm ζ(s) giải tích trên nửa mặt phẳng Re(s) > 1. Vậy ζ (s) = ζ(s) p (1/(1 − p−s )) =− 1/(1 − p−s ) p−s log(p) =− 1 − p−s p ∞ p log(p) . pms m=1 p−s log(p) log(p) Mà chuỗi và chuỗi hội tụ tuyệt đối khi Re(s) > 1. ms 1 − p−s p m=1 p Vậy, −ζ (s) log(p) = , ζ(s) pms ∞ trong đó tổng được sắp theo thứ tự tăng dần của pms . Suy ra, −ζ (s) = ζ(s) ∞ n=1 Ta kết thúc chứng minh. 61 Λ(n) . n Theo công thức tổng Abel, ta có x Λ(n) ψ(x) = +s n x n≤x 1 ψ(t) dt. ts+1 Từ đó, ta suy ra công thức rất quan trọng trong lý thuyết số giải tích sau −ζ (s) = ζ(s) 6 ∞ n=1 ∞ Λ(n) =s n 1 ψ(t) dt. ts+1 Một đánh giá cho mật độ số nguyên tố Định lý 9. lim inf x→∞ log(x)π(x) log(x)π(x) ≤ 1 ≤ lim sup ). x x x→∞ Chứng minh. Đặt π(x) log(x) ψ(x) = lim inf x→∞ x→∞ x x π(x) log(x) ψ(x) L = lim sup = lim sup x x x→∞ x→∞ −ζ (s) f (s) = ζ(s) l = lim inf (s − 1)f (s) l = lim inf x→1+ L = lim sup(s − 1)ζ(s). s→1+ Khi đó ta có f (s) = −ζ (s) =s ζ(s) ∞ 1 ψ(t) dt. ts+1 Xét B ∈ R sao cho L < B. Khi đó tồn tại R sao cho ψ(t) ≤ B, ∀t > R. t Từ đó ta có ∞ 1 ψ(t) = ts+1 R 1 ψ(t) dt + ts+1 62 ∞ R ψ(t) dt. ts+1 Bằng đánh giá cơ bản, ta có ∞ 1 ψ(t) B 1−s t ≤C+ s+1 t 1−s t→∞ . t=1 Ta nhận được (s − 1)f (s) ≤ (s − 1)sC + Bs. Ta suy ra ngay được L = lim sup(s − 1)f (s) ≤ B, ∀B > L. s→1+ Do đó L ≤ L. Tương tư ta cũng có l ≤ l . Kết hợp hai sự kiên này ta được l ≤ l ≤ L ≤ L. Do vậy để chứng minh lim inf x→∞ π(x) log(x) π(x) log(x) ≤ 1 ≤ lim sup . x x x→∞ ta chỉ cần chứng minh l = L = 1. Ta có f (s) = Mà ζ(s) = 1 s−1 −ζ (s) . ζ(s) + h(s) với h là hàm giải tích trên s > 1. Vậy ta có ζ (s) = −1 + h (s). (s − 1)2 Từ các sự kiện trên ta suy ra − lim (s − 1)f (s) = lim (s − 1) s→1+ s→1+ −1 (s−1)2 1 s−1 + h (s) + h(s) Giới hạn này có thể viết dưới dạng 1 − (s − 1)2 h (s) = 1. s→1+ 1 + (s − 1)h(s) lim Vậy l = L = lim (s − 1)f (s) = 1. Ta kết thúc chứng minh. s→1+ 63 . 7 Một số công thức của Mertens Định lý 10. Chúng ta có các kết quả sau. 1. n≤x 2. p≤x Λ(n) n = log(x) + O(1). log(p) = log(x) + O(1). p 3. x ψ(t) dt 1 t2 4. 1 p≤x p = log(x) + O(1). 1 = log log(x) + C + O( log(x) ), với C là một hằng số. Chứng minh. Bằng công thức tổng Abel, ta suy ra được m log(m!) = m log(m) − 1 [t] dt. t Theo bổ đề trong chứng minh định lý 3, ta suy ra được pvp , m! = p≤m với vp = ∞ s=1 m ps . Từ đó, ta suy ra log(m!) = vp log p = p≤m m Λn log p = m . s p n s p ≤m n≤m Từ đẳng thức trên, ta thu được Λ(n) 1 = log(m) − n m n≤m m 1 [t] dt. t Hay là Λ(n) 1 = log(x) + log [x] − log(x) − n [x] n≤x Rút gọn biểu thức trên ta thu được Λ(n) = log(x) + O(1) n n≤x 64 x 1 [t] dt. t khi x → ∞. Ta lại có log p = p Λn log(p) − . s n p s n≤x 2≤s;p ≤x Ta suy ra ngay được p≤x log p − log x = p Λn log(p) − log x − . n ps n≤x 2≤s;ps ≤x Theo đó p≤x log p − log x ≤ p Λn − log x + n n≤x s≥2 ps ≤x ≤M+ p≤x log p ps log p p2 − p 4 ≤M+ 3 p≤x ∞ ≤M+ p2 4 3 n=1 n2 khi x đủ lớn. Vậy p≤x log p = log x + O(1) p khi x → ∞. Để chứng minh (3) ta lại dùng công thức tổng Abel ψ(x) Λ(n) = + n x n≤x x 1 ψ(t) dt. ts Hơn nữa ψ(x) = O(x) nên ta suy ra được x 1 ψ(t) = t2 Λ(n) ψ(x) − = log(x) + O(1) + O(1) = log(x) + O(1), n x n≤x khi x đủ lớn. Đặt dãy (an ) với an = log(p) , p 0, nếu n = p trường hợp còn lại. 65 Khi đó dễ dàng tính được A(x) = p≤x log(p) . Đặt ϕ(x) = p dụng công thức tổng Abel ta suy ra được p≤x 1 = p p≤x log(p) p x 1 + log(x) p≤x 1 . log(x) Khi đó áp log(p) p dt. 2 t. log (t) 2 Tổng này có thể viết lại p≤x x 1 1 = 1 + O( )+ p log(x) (log(p)/p) − log(t) p≤x t(log(t))2 2 x dt + 2 1 dt. t log(t) Vậy p≤x 1 1 = 1 + O( )+ p log(x) ∞ − p≤x log(p) − p 2 ∞ 2 log(t) t. log (t) x p≤x log(p) − log(t) p dt 2 t.log (t) dt + log log x − log log 2. Đặt ∞ C =1+ p≤x 2 log(p) − log(t) p dt t.log 2 (t) − log log 2. Theo trên ta có p≤x 1 M − log log(x) − C ≤ + p log x ∞ | p≤x x log(p) − p 2 t. log (t) log(t)| dt ∞ M 1 +D dt log(x) t log2 (t) x M D ≤ + , log(x) log(x) ≤ khi x đủ lớn. Vậy 1 p≤x p 1 = log log(x) + C + O( log(x) ) khi x → ∞. TÀI LIỆU THAM KHẢO [1] K. Chandrasekharan, Introduction to Analytic Number Theory, Springer, 1968. 66 Định lý Weyl về phân bố đều và định lý Kronecker Người trình bày: Vũ Xuân Trường Trong bài giảng này, chúng ta sẽ giới thiệu về dãy số phân bố đều modulo 1 và chứng minh định lí của Herman Weyl về đặc trưng của những dãy số như vậy. Tìa liệu tham khảo là chương VIII [1]. 1 Tiêu chuẩn của Weyl về phân bố đều Định nghĩa 1. Cho dãy u1 , u2 . . . là một dãy các số thực. Ta nói dãy số này phân bố đều modulo 1 nếu với mọi đoạn (a, b) trong đoạn [0, 1] ta luôn có 1 |{{u1 }, {u2 }, . . . , {uN }} ∩ (a, b)| = b − a. N →∞ N lim Điều này có nghĩa là tỉ lệ các phần tử của dãy {uj } nằm trên một đoạn bất kì cho trước đúng bằng độ dài của đoạn đó. Hiển nhiên là dãy phần lẻ trù mật trong [0, 1]. Một chú ý nữa là ta có thể thay (a, b) bằng [a, b) vì sự sai khác giữa hai dãy 1 |{{u1 }, {u2 }, . . . , {uN } ∩ (a, b)| , N và 1 |{{u1 }, {u2 }, . . . , {uN } ∩ [a, b)| . N sẽ không còn khi chuyển qua giới hạn. Tương tự như vậy ta có thể thay (a, b) bằng bất kì đoạn có dạng [a, b) hoặc [a, b]. Định lí 1. Cho dãy số thực u1 , u2 , . . .. Khi đó các khẳng định sau là tương đương: 1. {ui } phân bố đều theo modulo 1. 67 2. Với mỗi k = 0 ta có 1 lim n→∞ n n e2πkuh = 0. h=1 3. Với mỗi hàm khả tích Riemann f : [0, 1] → C ta đều có n n→∞ 1 f ({uh }) = lim f (x)dx. 0 h=1 Chứng minh. Đầu tiên ta sẽ chứng minh sự tương đương của (1) và (3). Ta có thể giả sử rằng f là hàm nhận giá trị thực. Nếu f nhận giá trị phức, ta có thể xét các phần thực và phần ảo của hàm f . Ngoài ra ta cũng giả sử các số ui nằm trong đoạn [0, 1]. Giả sử (3) được thỏa mãn. Khi đó với mọi đoạn [a, b) nằm trong [0, 1], xét f là hàm đặc trưng của đoạn [a, b). Ta có 1 n n f (uh ) = h=1 ϕn (a, b) . n 1 f (x)dx = b − a. Theo điều kiện (3) ta có Mặt khác 0 ϕn (a, b) = b − a. n→∞ n lim Theo định nghĩa, dãy {uj } là một dãy có phân bố đều modulo 1. Giả sử ngược lại {ui } có phân bố đều. Khi đó đẳng thức điều kiện trong 1 đúng với các hàm đặc trưng f của mọi đoạn [a, b). Ý tưởng để chứng minh cho trường hợp f là hàm khả tích Riemann bất kì là xấp xỉ f bởi các hàm đơn giản. Do f là hàm khả tích Riemann trên [0, 1] nên với mọi > 0 ta có thể tìm được các hàm đơn giản f1 , f2 sao cho f1 ≤ f ≤ f2 và 1 (f2 (x) − f1 (x)) dx < . 0 Do đẳng thức (3) đúng cho f1 nên ta có 1 lim n→∞ n 1 n f1 (x)dx ≥ f1 (uh ) = h=1 1 0 f (x)dx − . 0 68 Với n đủ lớn ta có 1 n 1 n f (x)dx − 2 . f1 (uh ) > h=1 0 Do f ≥ f1 nên ta có 1 n 1 n f (x)dx − 2 . f (uh ) > h=1 0 Do đó khi n đủ lớn ta cũng có 1 n 1 n f (x)dx + 2 . f (uh ) < h=1 0 Từ hai bất đẳng thức ở trên ta suy ra được 1 n 1 n f (uh ) − h=1 f (x)dx < 2 , 0 khi n đủ lớn. Do đó đẳng thức (3) xảy ra với mọi hàm khả tích Riemann. Như vậy (1) và (3) tương đương. Ta sẽ chứng minh (2) tương đương (3). Hiển nhiên (3) suy ra (2) vì các hàm e2kπx khả tích. Khi đó, giả sử (3) đúng, áp dụng đẳng thức với f = e2πmx ta được 1 lim n→∞ n 1 n e2πkuh = h=1 e2kπx = 0. 0 Do đó (2) đúng. Giả sử ngược lại (2) đúng. Ta chứng minh (3) cũng đúng. Ý tưởng của chứng minh là xấp xỉ các hàm khả tích bằng các hàm lượng giác e2kπx với k ∈ Z và áp dụng giả thiết (2). Do các hàm lượng giác cơ bản cos(2kπx) và sin(2kπx) đều là tổng tuyến tính của các hàm lượng giác e2kπx nên (3) đúng với các hàm tổ hợp tuyến tính của các hàm lượng giác và hàm f = 1. Nghĩa là đẳng thức trong (3) đúng với tất cả các đa thức lượng giác có dạng a0 + a1 cos(2πx) + b1 sin(2πx) + · · · + am cos(2mπx) + bm sin(2mπx). 69 Ta biết rằng theo tiêu chuẩn Wierstrass mọi hàm f liên tục với chu kì 1 đều tồn tại một đa thức lượng giác f có dạng trên sao cho |f − f | < . Tương tự như trong chứng minh (1) và (3) tương đương, ta suy ra (3) xảy ra với các hàm f liên tục có chu kì 1. Với g là hàm đơn giản bất kì trên [0, 1] ta có thể tìm được hai hàm liên tục g1 , g2 trên [0, 1] sao cho g1 ≤ g ≤ g2 và 1 (g2 − g1 )dx < . 0 Bằng cách này, ta thấy rằng (3) đúng với mọi hàm g khả tích Riemann trên [0, 1]. Vậy ta đã chứng minh (2) suy ra (3). Định lý được chứng minh. Một trong những kết quả nổi tiếng suy ra từ tiêu chuẩn Weyl là. Định lí 2. Cho θ là một số vô tỉ. Khi đó dãy {nθ} phân bố đều modulo 1. Chứng minh. Ta sẽ chỉ ra dãy số này thỏa mãn điề kiện (2). Thực vậy với k là số nguyên khác 0 ta có 1 n n e2kπhθ = h=1 1 |e2kπθ − ek(n+1)πθ | . n 1 − e2kπθ Do θ là số vô tỉ nên 1 − e2kπθ = 0. Do đó ta dễ dàng có đánh giá 1 n n e2kπhθ ≤ h=1 2 . n|1 − e2πθ | Vế phải tiến tới 0 khi n → ∞ nên vế trái cũng tiến tới 0 khi n → ∞. Vậy dãy {nθ} thõa mãn điều kiện của tiêu chuẩn Weyl. Theo định lí 1, dãy này có phân bố đều modulo 1. Hệ quả 1. Nếu θ là số vô tỉ thì dãy phần lẻ ({nθ}) trù mật khắp nơi trong đoạn thẳng đơn vị. Khái niệm phân bố đều có thể được mở rộng lên không gian với số chiều lớn hơn 1. Gọi (P (j) ) là một dãy vô hạn các điểm trong không gian Euclide p chiều. Giả sử rằng tọa độ của P (j) là (xj1 , . . . , xjp ). Ta gọi αjr là phần lẻ của xjr và {P (j) } là dãy các phần lẻ tương ứng ({xj1 }, . . . , {xjp }. Khi đó {P (j) } nằm trong hình lập phương đơn vị xác định bởi 0 ≤ xj < 1, ∀j ∈ {1, . . . , p}. Gọi V là một hình hộp nằm trong hình lập phương đơn vị và ta kí hiệu |V | 70 là độ đo Lebesgue của V . Ta nói dãy {P (j) } phân bố đều theo modulo 1 nếu dãy phần lẻ tương ứng phân bố đều trong hình lập phương đơn vị, nghĩa là với mọi hình hộp chữ nhật V nằm trong hình lập phương đơn vị, ta đều có ϕn (V ) = |V |, n→∞ n lim trong đó ϕn (V ) đếm số điểm trong n điểm đầu tiên của dãy nằm trong hình hộp chữ nhật V . Tương tự như trong trường hợp 1 chiều, ta cũng có tiêu chuẩn sau. Định lí 3. Dãy {P (j) } phân bố đều trong hình lập phương đơn vị nếu và chỉ nếu n 1 e2πi(m1 αh1 +···+mp αhp ) = 0, lim n→∞ n h=1 với mọi tập (m1 , . . . , mp ) = (0, . . . , 0). Chứng minh. Chứng minh của định lí này giống hệt trong trường hợp một biến. Ý tưởng chính vẫn là xấp xỉ các hàm đơn giản và sử dụng tính hội tụ của chuối Fourier. Một hệ quả trực tiếp của định lí trên là định lí sau đây. Định lí 4. Giả sử θ1 , . . . , θp là các số thực sao cho θ1 , . . . , θp , 1 độc lập tuyến tính trên Z. Khi đó dãy nθ = (nθ1 , . . . , nθp ) phân bố đều modulo 1. 2 Định lí Kronecker Định lí 4 chỉ ra rằng dãy {nθ} = ({nθ1 }, . . . , {nθp }) trù mật trong hình lập phương đơn vị. Đây chính là nội dung của định lí Kronecker. Ta nêu lại định lí. Định lí 5 (Kronecker). Giả sử θ1 , . . . , θk , 1 là các số thực và độc lập tuyến tính trên Z, α1 , . . . , αk là các số thực và N , là các số thực dương. Khi đó tồn tại các số nguyên n và p1 , . . . , pk sao cho n > N, |nθm − pm − αm | < , với mọi m = 1, 2, . . . , k. Ta đưa ra một phiên bản khác của định lí này. 71 Định lí 6. Nếu θ1 , . . . , θk là các số thực và độc lập tuyến tính trên Z và α1 , . . . , αk là các số thực tùy ý. Giả sử T và là các số thực dương. Khi đó tồn tại một số thực t và các số nguyên p1 , . . . , pk sao cho. t > T, |tθm − pm − αm | ≤ , với mọi m = 1, 2, . . . , k. Ta sẽ chứng minh rằng thực ra định lí 5 và định lí 6 tương đương. Đầu tiên ta giả sử rằng định lí 6 đúng, ta sẽ chỉ ra rằng định lí 5 suy ra từ đó. Ta hoàn toàn có thể giả sử rằng 0 < θm ≤ 1 với mọi m = 1, 2, . . . , k. Thực vậy, nếu 1, θ1 , . . . , θk độc lập tuyến tính trên Z thì các số 1, θ1 , . . . , θk cũng độc lập tuyến tính trên Z với θi = θi − qi với qi nguyên. Bất đẳng thức |nθm − pm − αm | < sẽ kéo theo |nθm − pm − αm | < với pm = pm + nqm . Do đó ta hoàn toàn có thể giả sử 0 < θm ≤ 1 với 1 ≤ m ≤ k, 0 < < 1 và 1, θ1 , . . . , θk độc lập tuyến tính trên Z. Áp dụng định lí 6 với tập θ1 , . . . , θk , 1; α1 , . . . , αk , 0, tồn tại p1 , . . . , pk+1 và t > N + 1 sao cho |tθm − pm − αm | < , 2 với m = 1, 2, . . . , k. Ngoài ra t cũng thỏa mãn |t − pk+1 | < . 2 Từ sự kiện trên ta có pk+1 > t − 0 < θm ≤ 1 ta có 1 2 > N do t > N + 1 và < 1. Ngoài ra, do |pk+1 θm − pm − αm | ≤ |tθm − pm − αm | + |(pk+1 − t)θm | < 2 + 2 = , với mọi m = 1, 2, . . . , k. Do đó định lí 5 được chứng minh với n = pk+1 . Ngược lại giả sử rằng định lí 5 đã được chứng minh, ta sẽ chứng minh định lí 6. Trong trường hợp k = 1 thì định lí 6 là tầm thường. Thực vậy nếu θ1 là số hữu tỉ ta có thể dễ dàng chọn t và pm . Trong trường hợp θ1 là số vô tỉ, đây chính là nội dụng của định lí 5. Do đó ta có thể giả sử k > 1. Ngoài ra ta cũng giả sử θm > 0 với m = 1, 2, . . . , k. Giả sử các số θ1 , . . . , θk độc lập tuyến tính trên Z, khi đó các số θ1 θk−1 ,..., , 1, θk θk 72 cũng độc lập tuyến tính trên Z. Nếu ta áp dụng định lí 5 với N = T θk cho tập hợp θk−1 θ1 ,..., ; α1 , . . . , αk−1 , θk θk ta tìm được các số p1 , . . . , pk−1 , n sao cho n > N thỏa mãn n Nếu ta đặt t = n θk θm − p m − αm < , θk m = 1, 2, . . . , k − 1. thì t > T và |tθm − pm − αm | < , m = 1, 2, . . . , k − 1 và hiển nhiên |tθk − n| < do đó ta có kết luận cho định lí 8 với tập hợp θ1 , . . . , θk ; α1 , . . . , αk−1 , 0. Tương tự ta cũng có thể chứng minh cho định lí 6 với tập θ1 , . . . , θ k ; 0, . . . , 0, αk , Hai khẳng định này dễ dàng cho ta kết luận của định lí 6. Ta sẽ đưa ra một chứng minh vô cùng đẹp đẽ của định lí 6 dựa trên chứng minh của H. BOHR. Chứng minh. Với c thực, T > 0 và i2 = −1 ta có T 0, nếu c = 0 1, nếu c = 0. ecit dt = lim T →∞ 0 Do đó nếu cv thực đôi một phân biệt và r bv ecv it , χ(t) = (1) v=1 thì T χ(t)e−cv it dt = bv . lim T →∞ 0 73 Xét k e2πi(tθm −αm ) , F (t) = 1 + (2) m=1 với t thực và ϕ(t) = |F (t)|, thì hiển nhiên ta có 0 ≤ ϕ(t) ≤ k + 1. Nếu định lí 6 là đúng thì với t đủ lớn nào đó, các số tθm − αm gần một số nguyên và do đó ϕ(t) gần k + 1. Ngược lại nếu ϕ(t) gần k + 1 với t đủ lớn thì mọi số hạng của dãy (2) cũng phải gần 1. Do đó định lí 6 theo nghĩa nào đó được chứng minh. Điều này có thể được chứng minh một cách chặt chẽ như sau. Giả sử tồn tại η với 0 < η < 1 sao cho ϕ(t) ≥ k + 1 − η và z = e2πim = x + iy. Khi đó ta có k + 1 − η ≤ ϕ(t) ≤ (k − 1) + |1 + e2πixm |, hay là 2 ≥ |1 + e2πixm | ≥ 2 − η, với m = 1, 2, . . . , k. Ta lại có |1 + z|2 = (1 + x)2 + y 2 = (1 + x)2 + (1 − x2 ) = 2 + 2x ≥ (2 − η)2 > 4 − 4η, do đó ta có 1 ≥ x ≥ 1 − 2η. Tổng hợp lại ta có y 2 = 1 − x2 = (1 − x)(1 + x) ≤ 2(1 − x) ≤ 4η, √ √ hay là |y| ≤ 2 η. Từ đó |z − 1| < 4 η. Từ những nhận xét trên ta thấy rằng, định lí 6 sẽ được chứng minh nếu ta chứng minh được rằng lim sup ϕ(t) ≥ k + 1. t→∞ Đặt ψ = ψ(x1 , . . . , xk ) = 1 + x1 + · · · + xk , và p là một số nguyên dương. Khi đó an1 ,...,nk xn1 · · · xnk k , ψp = (3) n1 +···+nk ≤p nj ≥0 trong đó các hệ số an1 ,...,nk có các tính chất đơn giản sau 74 1. Các hệ số này đều dương 2. an1 ,...,nk = ψ p (1, . . . , 1) = (k + 1)p 3. Có nhiều nhất là (p + 1)k các hệ số. Ta sẽ sử dụng các tính chất này để xét tổng p k p F (t) = 2πi(tθm −αm ) 1+ e . m=1 Áp dụng các nhận xét trên với xj = e2πi(tθj −αi ) , ta thấy ngay được rằng F p (t) là tổng của các hạng tử có dạng như trong (1), với cv = 2π(n1 θ1 + · · · + nk θk ). Do các số θ là độc lập tuyến tính trên Z các số hạng cv đôi một phân biệt. Ngoài ra ta cũng dễ dàng nhận thấy bv = e−2πi(n1 α1 +···+nk αk ) . Do đó ta có |bv | = an1 ,...,nk = (k + 1)p . Do ϕ(t) ≤ k + 1, để chứng minh lim supt→∞ ϕ(t) ≥ k + 1 ta chỉ cần chứng minh lim sup ϕ(t) > k + 1, t→∞ là không thể xảy ra. Giả sử ngược lại, khi đó ta có |F (t)| = ϕ(t) ≤ λ < k + 1, với t đủ lớn. Do đó T 1 lim sup T →∞ T T 1 |F (t)| dt ≤ lim T →∞ T p 0 λp = λp . 0 Tuy nhiên, 1 F (t)p e−cv it dt, T →∞ T bv = lim và do đó T |F (t)|p dt ≤ λp . |bv | ≤ lim sup T →∞ 0 75 Do đó tất cả các hệ số trong (3) thỏa mãn an1 ,...,nk ≤ λp . Do có nhiều nhất là (k + 1)p số hạng, ta có (k + 1)p = an1 ,...,nk ≤ (p + 1)k λp . (4) p λ λ Ta biết rằng, do k+1 < 1 nên limp→∞ k+1 (p + 1)k → 0. Từ đó bất đẳng thức 4 không thể xảy ra. Ta có điều phải chứng minh. TÀI LIỆU THAM KHẢO [1] K. Chandrasekharan, Introduction to Analytic Number Theory, Springer, 1968. 76 Định lí Minkowski về điểm nguyên trong một tập lồi Người trình bày: Vũ Xuân Trường Ở chương này ta sẽ chứng minh định lí Minkowski về tồn tại điểm nguyên trong tập lồi. Tài liệu tham khảo là chương IX[1]. 1 Tập lồi Ở chương VI, ta đã làm quen với một số bài toán về số điểm nguyên trong một miền trên mặt phẳng. Ta kí hiệu Rn là không gian Euclide n chiều. Ta gọi một điểm là điểm nguyên nếu các tọa độ của nó là các số nguyên. Định nghĩa 1. Cho S là một tập trong Rn và λ là một số thực. Ta định nghĩa tập λS là tập hợp gồm những số thực có dạng λs với s ∈ S. Ta gọi S là lồi nếu với mọi x, y ∈ S và λ ∈ (0, 1) thì λx + (1 − λ)y cũng thuộc S. Một nhận xét đơn giản là phép vị tự x → λx bảo toàn tính lồi. Nói cách khác, nếu S là tập lồi thì λS cũng là tập lồi. Ta gọi tập S là đối xứng qua gốc tọa độ nếu x ∈ S thì ta cũng có −x ∈ S. Tương tự như ở trên, nếu S đối xứng thì λS cũng đối xứng. Giả sử g là một điểm nguyên, ta gọi Sg là ảnh của S qua phép tính tiến theo vector g. Nói cách khác Sg = {x + g | x ∈ S}. Nếu S là tập đo được Lebesgue với độ đo V (S) thì ta có V (S) = V (Sg ) với mọi điểm nguyên g. Đối với mọi tập lồi và đối xứng ta có: 1. Nếu S là tập lồi và đối xứng thì với mọi x ∈ S thì λx ∈ S với mọi |λ| ≤ 1. Thật vậy, nếu λ ≥ 0 thì ta có λx = λx + (1 − λ)0. Do x, 0 ∈ S nên ta λx cũng thuộc S. Nếu λ < 0 thì −λx ∈ S theo lập luận trên. Do S đối xứng nên λx cũng thuộc S. Vậy λx ∈ S với mọi |λ| ≤ 1. 77 2. Nếu S là tập lồi và đối xứng thì với mọi x, y ∈ S và λ, µ sao cho |λ| + |µ| ≤ 1 ta cũng có λx + µy ∈ S. Nếu µ hoặc λ bằng 0, ta quay lại trường hợp đầu tiên. Do đó ta có thể giả sử λ, µ đều khác 0. Nói chung ta có thể giả sử λ, µ > 0. Nhận xét này được suy ra từ tính chất λx = −λ(−x) và tính đối xứng của S. 2 Định lí Minkowski Định lí 1 (Minkowski). Mọi tập con của Rn lồi, bị chặn, đối xứng qua gốc tọa độ và đo được với độ đo lớn hớn 2n chứa ít nhất một điểm nguyên khác gốc tọa độ. Ta sẽ đưa ra một chứng minh dựa theo chứng minh của C.L Siegel. Chứng minh này dựa trên công thức cho độ đo của một tập bị chặn, đo được, lồi , đối xứng và không chứa một điểm nguyên nào ngoài gốc tọa độ. Giả thiết về tính bị chặn của tập S là không thực sự cần thiết. Ta sẽ chỉ ra điều này trong định lí 3 dưới đây. Chứng minh. Chứng minh bằng phản chứng. Gọi S một tập con của Rn bị chặn, lồi, đối xứng qua gốc tọa độ và đo được với độ đo V và kí hiệu L2 (S) là tập hợp các hàm khả tích chính phương trên S. Nói cách khác |f |2 dµ < ∞}. L2 (S) = {f đo được | S Với một hàm ϕ ∈ L2 (S) ta có thể mở rộng miền xác định của nó lên toàn bộ không gian bằng cách đặt ϕ(x) = 0 nếu x ∈ S. Ta viết k = (k1 , . . . , kn ), x = (x1 , . . . , xn ). Tích vô hướng kx = k1 x1 + · · · + kn xn . Ngoài ra ta cũng kí hiệu dx = dx1 · · · dxn . Xét hàm số f (x) = k ϕ(2x − 2k) (∗) với k chạy trên tập các điểm nguyên trên mặt phẳng. Với mỗi x cố định thì tổng này hữu hạn do ϕ triệt tiêu ngoài S và tập S bị chặn. Do tổng này xác định trên toàn bộ lưới nguyên nên tổng không đổi nếu ta thay đổi kv → kv + 1. Do đó hàm f tuần hoàn theo từng biến x1 , . . . , xn với chu kì 1. Đẳng thức Parseval cho chuỗi Fourier của f cho ta kết quả |f |2 dx = E |al |2 , l trong đó E là hình vuông chiều n với độ dài 1, l là điểm nguyên trong Rn và al là hệ số Fourier của f xác định bởi f (x)e−2πilx dx. al = E 78 Theo công thức xác định f ta có ϕ(2x − 2k)e−2πilx dx = al = E k ϕ(2x − 2k)e−2πilx dx. E k Đặt x − k = t thì khi x chạy trên E và k chạy trên tất cả tập điểm nguyên thì t chạy trên toàn bộ Rn . Do đó ta có ϕ(2t)e−2πil(k+t) dx = al = ϕ(2t)e−2πit dt. Rn Rn Do đó nếu ta đổi biến 2t = x ta nhận được al = 2−n ϕ(x)e−πilx dx. S Mặt khác từ công thức xác định f ở (∗) ta có |f |2 dx = ϕ(2x−2k)ϕ(2x − 2k )dx = ( E k E ϕ(2x−2k)ϕ(2x)dx. Rn k k Tích phân này có thể được tính tiếp |f |2 dx = 2−n ϕ(x − 2k)ϕ(x)dx = 2−n Rn E k ϕ(x − 2k)ϕ(x)dx. k S Từ đẳng thức Parseval ta có ϕ(x − 2k)ϕ(x)dx = 2−n S k ϕ(x)e−πilx |2 dx. | l S Nếu ϕ(x) và ϕ(x − 2k) khác 0 thì ta phải có x, x − 2k ∈ S. Khi đó k = 1 x + 12 (2k − x) cũng thuộc S. Điều này là không thể do S không chứa điểm 2 nguyên nào. Do đó ϕ(x − 2k)ϕ(x) = 0 với k = 0. Do đó đẳng thức trên có thể viết lại |ϕ(x)|2 dx = 2−n S ϕ(x)e−πilx dx|2 , | l S Bây giờ nếu ta xác định ϕ(x) = 1 với mọi x ∈ S thì ta có |f |2 dx = V. S 79 (∗∗) Đẳng thức (∗∗) có thể viết lại V = 2−n e−πilx dx|2 = 2−n V 2 + | l S e−πilx dx|2 | l=0 . S Ta có thể viết lại công thức này dưới dạng 2n = V + 1 V eπilx dx|2 , | l=0 (8) S Đây chính là công thức Siegel cho độ đo V của một tập đo được bị chặn, lồi, đối xứng và không chứa điểm nguyên nào ngoài gốc tọa độ. Từ công thức trên ta có ngay được V ≤ 2n . Điều mâu thuẫn này chứng tỏ S chứa ít nhất một điểm nguyên khác gốc tọa đô và đo đó định lí 1 được chứng minh. Nếu ta chỉ muốn chứng minh định lí Minkowski mà không cần công thức (8) thì có thể sử dụng bất đẳng thức Schwartz |f |dx ≥ |a0 |2 . E Ta cũng có ϕ(x)dx = 2−n V. a0 = 2−n S Từ bất đẳng thức Schwarz ta nhận được |f |2 dx ≥ 2−n V. E n Từ đó nhận được V ≤ 2 . Định lí 1 không còn đúng nếu V = 2n . Một ví dụ cụ thể là tập |xi | < 1, ∀i ∈ {1, . . . , n}. Tập này bị chặn, đo được, lồi, đối xứng nhưng không chứa bất kì điểm nguyên nào khác ngoài gốc tọa độ. Tuy nhiên trong trường hợp S là tập đóng, ta có khẳng định sau. Định lí 2. Một tập compact, lồi, đối xứng S trong Rn với độ đo V (S) ≥ 2n chứa một điểm nguyên ngoài gốc tọa độ. Chứng minh. Với mỗi 0 < < 1 ta xét tập S = (1 + )S. Do S đo được nên S cũng đo được và hơn nữa V (S ) = (1 + )n V (S) ≥ (1 + )n 2n > 2n Do đó theo định lí 1, S chứa một điểm nguyên khác gốc tọa độ. Tập 2S chỉ có hữu hạn điểm nguyên do nó bị chặn. Hơn nữa với 0 < < δ < 1 ta có S ⊂ Sδ ⊂ S1 . Do đó dễ thấy tồn tại một điểm nguyên l sao cho l thuộc S với mọi > 0. Lấy → 0 và do S đóng ta suy ngay được l ∈ S. Đó là điều phải chứng minh. 80 Một hệ quả trực tiếp của định lí 2 là định lí sau. Định lí 3. Nếu S là tập bị chặn, lồi, đối xứng và có độ đó ít nhất là 2n . Khi đó S¯ chứa một điểm nguyên khác gốc tọa độ. ¯ Khi đó ta cần Chứng minh. Do S lồi nên S¯ cũng lồi. Thật vậy, xét x, y ∈ S. ¯ Do chứng minh rằng với mọi λ ∈ (0, 1) thì z = λx + (1 − λ)y cũng thuộc S. x, y ∈ S¯ nên tồn tại hai dãy {xk }, {yk } trong S sao cho lim xk = x, lim yk = y. k→∞ k→∞ Khi đó xét xãy zk = λxk + (1 − λ)yk . Ta có, theo bất đẳng thức tam giác |z − zk | ≤ λ|x − xk | + (1 − λ)|y − yk |. ¯ Một cách Hệ quả của bất đẳng thức này là zk → z. Theo định nghĩa z ∈ S. ¯ tương tự ta cũng chứng minh được S là tập đối xứng, bị chặn. Ngoài ra ta ¯ ≥ V (S) ≥ 2n . Theo định lí 2, S¯ chứa một điểm nguyên khác gốc có V (S) tọa độ. Để chứng minh định lí Minkowski ta cần bổ đề sau. Bổ đề 1 (Birkhoff). Nếu S là tập đo được trong Rn với độ đo V (S) > 1. Khi đó tồn tại hai điểm x, y ∈ S sao cho x − y là một điểm nguyên. Chứng minh. Với mỗi điểm nguyên g ta xét tập: Tg = {x|gi ≤ xi < gi + 1}. Đặt S g = S Tg . Ảnh của S g qua phép tịnh tiến theo vector −g được kí g g hiệu là S−g . Hiển nhiên ta có S−g nằm trong hình vuông đơn vị C = {x|0 ≤ g xi < 1, ∀i}. Nếu ta kí hiệu Vg là độ đo của tập S−g thì do Sg, S= g nên ta có: g V (S g ) = V (S) > 1. Phép tịnh tiến bảo toàn độ đo nên ta cũng có g Vg = V (S) > 1. Mặt khác do độ đo của tập C bằng 1 nên tồn tại hai g h tập S−g và S−h có giao khác rỗng. Tức là, tồn tại x ∈ S g và y ∈ S h sao cho x − g = y − h. Nói cách khác x − y = g − h. Do đó x − y là một điểm nguyên. Bổ đề được chứng minh xong. Bây giờ, ta có thể chứng minh định lí Minkowski dạng tổng quát. 81 Định lí 4 (Minkowski). Nếu S là tập lồi, đối xứng và đo được với độ đó V (S) > 2n thì nó chứa một điểm nguyên khác gốc tọa độ. Chứng minh. Xét tập 21 S. Tập này có độ đo là ( 12 )n V > 1. Theo định lí Birkhoff tồn tại x, y ∈ 12 S sao cho x − y là một điểm nguyên. Tập 12 S lồi và đối xứng, do đó ta có 21 x − 21 y = 21 g ∈ S. Từ đó g ∈ S. Hiển nhiên g không là gốc tọa độ do x, y phân biệt. Định lí được chứng minh. Các định lí trên có thể áp dụng vào các dạng tuyến tính thuần nhất. Cụ thể như sau. Giả sử n aij xj , ∀i ∈ {1, . . . , n}, vi = j=1 với các hệ số aij thực. Gọi ∆ là định thức của ma trận (aij ). Ta giả sử rằng ∆ = 0. Các dạng tuyến tính thuần nhất này xác định một ánh xạ tuyến tính của x-không gian sang v-không gian. Do ánh xạ tuyến tính bảo toàn tính lồi và đối xứng, tập ảnh T của S cũng lồi và đối xứng. Độ đo của S thay đổi với hệ số |∆|−1 theo công thức đổi biến dv1 . . . vn = ∆ T dx1 . . . xn . S Xét ánh xạ tuyến tính L của Rn vào chính nó, cho bởi (x1 , . . . , xn ) → (v1 , . . . , vn ). Ảnh của tập các điểm với tọa độ nguyên được gọi là một lưới nguyên Λ xác định bởi L. Định thức của L được gọi là định thức của lưới Λ. Một ứng dụng của định lí 1 cho v-không gian là định lí sau đây. Định lí 5. Nếu Λ là một lưới với định thức ∆ > 0 và P là một tập đo được, đối xứng với độ đo V > 2n |∆| thì P chứa một điểm khác gốc tọa độ của Λ. Một ứng dụng của định lí 2 là. Định lí 6. Giả sử Λ là môt lưới với định thức khác 0, và P là một tập đóng, bị chặn, đối xứng với độ đo V ≥ 2n |∆| thì P chứa một điểm khác gốc tọa độ của lưới Λ. 3 Ứng dụng của định lí Minkowski 1. Xét một tập S trong x-không gian xác định bởi |vi | ≤ ci , i = 1, . . . , n. 82 Tập S hiển nhiên là tập đối xứng và lồi và bị chặn do nó là ảnh của tập T trong v-không gian xác bởi |vi | ≤ ci qua ánh xạ tuyến tính L−1 . Ta hiển nhiên cũng có V (S) = |∆|−1 V (T ) = 2n |∆|−1 c1 · · · cn . Áp dụng định lí 6 ta được. Định lí 7. Nếu v1 , . . . , vn là các dạng tuyến tính thuần nhất theo biến x1 , . . . , xn với các hệ số thực và định thức khác 0, ci là các số thực dương sao cho c1 , . . . , cn > |∆| thì luôn tồn tại các số nguyên x1 , . . . , xn , không đồng thời bằng 0 sao cho |vi | ≤ ci , ∀i ∈ {1, . . . , n}. 1 Tất nhiên ta có thể chọn ci = |∆| n với mọi i. Chúng ta đã giả sử ∆ = 0. Nếu ∆ = 0 ta dễ dàng thấy rằng tập S có độ đo vô hạn nếu ci > 0. Khi đó định lí 5 vẫn đúng. Do đó thay vì xét chặn cho mọi vi ta giảm số bất đẳng thức xuống. Cụ thể, ta giả sử rằng n |vi | = | aij xj | ≤ ci , i = 1, ..., m, m < n. (12) j=1 Khi đó tập S không bị chặn. Tuy nhiên kết luận trong định lí 7 vẫn xảy ra theo định lí 4. Ngoài ra ta cũng chú ý rằng trường hợp m < n được đưa về trường hợp m = n, ∆ = 0 bằng cách viết lại (12) n − m + 1 lần. 2. Ứng dụng thứ 2 là xét tập T trong v-không gian cho bởi |v1 | + · · · + |vn | ≤ c. Tập này hiển nhiên đối xứng, bị chặn. Ngoài ra ta nhận xét rằng nếu (s1 , . . . , sn ) ∈ T và (t1 , . . . , tn ) ∈ T và λ ∈ (0, 1) thì ta có n n |λsi + (1 − λ)ti | ≤ k=1 (λ|si | + (1 − λ)|ti |) = c. k=1 Nói cách khác, tập T là tập lồi. Thể tích của T có thể được tính như sau. T chia thành 2n tập con có cùng thể tích, mỗi tập nằm trong một miền và có cùng thể tích với thành phần v1 > 0, . . . , vn > 0. Phần này có thể tích 1 cn 0 1−v1 −···−vn−1 1−v1 dv1 dv2 . . . 0 Do đó T có thể tích V = ta. dvn = 0 (2c)n n! cn n! . Nếu ta chọn cn ≥ n!|∆|, định lí 6 cho 83 Định lí 8. Tồn tại các số nguyên x1 , . . . , xn không đồng thời bằng 0 sao cho 1 |v1 | + · · · + |vn | ≤ (n!|∆|) n . Do |v1 · · · vn | ≤ n1 (|v1 | + · · · + |vn |) nên ta có định lí sau. Định lí 9. Tồn tại các số nguyên x1 , . . . , xn không đồng thời bằng 0 sao cho n!|∆| . |v1 · · · vn | ≤ nn 3. Xét tập P trong v- không gian xác định bởi v12 + · · · + vn2 ≤ c2 . Tập này hiển nhiên đối xứng và lồi. Điều này có thể dễ thấy dựa trên bất đẳng thức Minkowski  2 n n 2 k=1 n tk 2  . 2 (λsk + µtk ) ≤ λ sk + µ k=1 k=1 Bằng quy nạp ta dễ dàng chứng minh được thể tích của P là n c n cn π 2 dv1 · · · dvn = n = cn s n , Γ( 2 + 1) Sn với Sn là quả bóng n chiều. Do đó nếu ta chọn c ≥ 2( s∆n ) ta nhận được. Định lí 10. Tồn tại các số nguyên x1 , . . . , xn không đồng thời bằng 0 sao cho |∆| 2 |v1 |2 + · · · + |vn |2 ≤ 4( )n . sn Định lí này có thể mở rộng cho các dạng toàn phương xác định dương n Q(x1 , . . . , xn ) = ars xr ss , r,s=1 với ars = asr . Định thức D của ma trận (ars ) được gọi là định thức của dạng toàn phương. Do Q xác định dương D > 0. Ta biết rằng mọi dạng toàn phương xác định dương có thể biểu diễn dưới dạng Q = v12 + · · · + vn2 , trong đó v1 , . . . , vn là các dạng tuyến tính của x1 , . . . , xn với định thức √ D. Định lí 7 do đó có thể được viết là. 84 Định lí 11. Nếu Q là một dạng toàn phương xác định dương với định thức D thì tồn tại các số nguyên x1 , . . . , xn không đồng thời bằng 0 sao cho D 1 Q(x1 , . . . , xn ) ≤ 4( 2 ) n , sn n trong đó sn = π2 . Γ( n +1) 2 TÀI LIỆU THAM KHẢO [1] K. Chandrasekharan, Introduction to Analytic Number Theory, Springer, 1968. 85 Định lí Dirichlet về số nguyên tố trong một cấp số cộng Người trình bày: Nguyễn Thọ Tùng 1 Lời tựa Phần trình bày của chương này chủ yếu dựa vào hai tài liệu [1], [2]. Như đã đã biết, bằng các lập luận khá sơ cấp ở các chương trước, ta đã chỉ ra rằng tồn tại vô hạn số nguyên tố có dạng 4k + 1 và 4k + 3. Mục đích của chương này là chứng minh một định lí tổng quát hơn, định lí Dirichlet về sự tồn tại vô hạn số nguyên tố trong một cấp số cộng a + mk với a, m là các số nguyên, m > 0, gcd(a, m) = 1 và k nhận các giá trị nguyên dương. 1 phân kì với p chạy trong Trong chương 7, ta đã chứng minh chuỗi p tập hợp tất cả các số nguyên tố. Chứng minh có thể tóm lược lại như sau. Với s thực và lớn hơn 1, ta có đẳng thứ Euler ∞ 1 1 (1 − s )−1 . = s n p p∈P ζ(s) = i=1 Với 0 < x < 1, theo công thức khai triển Taylor, ta có 1 log( )= 1−x Do đó với 0 < x ≤ 1 2 ∞ n=1 xn < n ∞ xn = n=1 ta có log( 1 ) < 2x. 1−x Áp dụng bất đẳng thức này với x = log(1 − 1 ps ta có 1 −1 2 ) < s. s p p 86 x . 1−x Từ đó ta thu được log ζ(s) = log(1 − 1 −1 ) 1. Giả sử p p1 hội tụ, ta suy ra ngay được là 2 p p−s < 2 p p1 . Tuy nhiên ta biết rằng ζ(1 + ) → ∞ khi → 0. Do đó p p1 phân kì. Tương tự như lập như lập luận ở trên về sự liên quan giữa sự phân kì 1 p p của chuỗi p≡a (mod m) 1 p ∞ và tính chất của hàm ζ(s) = n=1 ∞ liên quan tới tính chất của chuỗi n=1 n−s , sự phân kì của chuỗi an ns trong đó s và các hệ số an đều là các số phức. Ta sẽ nghiên cứu sự liên hệ này bằng cách xét hàm ζ(s) với giá trị phức của s. Đặt s = σ + it với σ, t là các số thực. Với σ > 1 ta có | n1s | = n1σ . Do đó ta có ∞ ∞ 1 1 = . s |n | n=1 nσ n=1 1 Từ đó chuỗi ∞ n=1 ns hội tụ tuyệt đối với σ > 1 và hội tụ đều trên nửa mặt phẳng σ ≥ 1 + δ với mọi δ > 0. Do đó với tổng chuỗi là một hàm chỉnh hình trên nửa mặt phẳng σ > 1. Theo chương VII, ta có đẳng thức ∞ ζ(s) = n=1 1 = ns (1 − p 1 −1 ) . ps trong nửa mặt phẳng σ > 1. Ta sẽ chứng minh rằng hàm ζ(s) xác định như trên có thể thác triển thành một hàm chỉnh hình trên nửa mặt phẳng σ > 0 ngoại trừ đúng một điểm cực đơn với thặng dư là 1 tại điểm s = 1. Để chứng minh điều này ta sử dụng công thức tổng Abel với λn = n, ϕ(x) = x−s và an = 1. Khi đó A(x) = [x] và [x] 1 = s +s s n x n≤x x 1 [u] du. us+1 Ta có [x] → 0 khi x → ∞ khi σ > 1. Khi đó chuỗi xs viết [u] = u − {u} ta có biểu diễn ∞ n=1 1 =s ns ∞ 1 du −s u ∞ 1 ∞ 1 n=1 ns {u}du s = −s s+1 u s−1 87 ∞ 1 hội tụ. Nếu ta {u}du . us+1 Do đó với σ > 1 ta có ∞ n=1 1 1 −s =1+ s n s−1 ∞ 1 {u}du , us+1 (σ > 1). Ta có 0 ≤ {u} < 1, từ đó ta nhận thấy tích phân trên hội tụ tuyệt đối và đều trên nửa mặt phẳng σ ≥ δ > 0 và biểu diễn một hàm chỉnh hình của s với σ > 0. Ta kết luận rằng hàm ζ(s) có thể thác triển thành một hàm phân hình trên nửa mặt phẳng σ > 0 với một cực điểm đơn duy nhất tại s = 1 với thặng dư là 1. Hàm này được gọi là hàm Zeta Riemann. 2 Đặc trưng Cho G là một nhóm Abel. Một đặc trưng của G là một đồng cấu nhóm của G vào C∗ . Nói một cách khác, χ là một đặc trưng của G nếu với mọi A,B trong G ta đều có χ(AB) = χ(A)χ(B). Gọi E là phần tử đơn vị của G và A−1 là phần tử nghịch đảo của A trong G. Khi đó ta có các tính chất sau 1. χ(E) = 1. 2. Giả sử G có bậc h, khi đó theo định lí Lagrange ta có Ah = E với mọi A trong G. Từ đó ta có χ(A)h = χ(Ah ) = χ(E) = 1. Do đó χ(A) là một căn bậc h của đơn vị với mọi A trong G. Với mỗi nhóm G ta luôn có một đặc trưng tầm thường xác định bởi χ(A) = 1 với mọi A trong G. 3. Một nhóm G có bậc h thì có đúng h đặc trưng. Trước hết ta chứng minh khẳng định này trong trường hợp G la một nhóm cyclic. Giả sử rằng G =< A > và |G| = r. Khi đó một đồng cấu χ từ G vào C∗ được xác định duy nhất theo χ(A). Theo nhận xét 2, χ(A) là một căn r của đơn vị. Hiển nhiên với mỗi ρ là một căn r của đơn vị, ta xác định một đồng cấu nhóm duy nhất từ G vào C∗ cho bởi: χ(Aj ) = ρj , j ∈ {0, . . . r − 1}. Do có đúng r căn của đơn vị nên số cách chọn ρ chính bằng r. Theo nhận xét ở trên, trong trường hợp G là một nhóm cyclic, G có đúng |G| đặc trưng. 88 Để chứng minh định lí tổng quát, ta cần một kết quả về cấu trúc của các nhóm Abel hữu hạn sinh. Cụ thể như sau: Mọi nhóm Abel hữu hạn đều là tích của các nhóm cyclic. Ta có G∼ = G1 × G2 × · · · × Gk . Giả sử rằng cấp của Gi là ri và Gi =< Ai > với mọi i ∈ {1, . . . , k}. Khi đó mọi phần tử trong G có thể viết dưới dạng At11 · · · Atkk với 0 ≤ ti ≤ ri − 1, ∀i ∈ {1, . . . , k}. Với mỗi đặc trưng χ ta có χ(At11 · · · Atkk ) = χ(A1 )t1 · · · χ(Ak )tk . Từ đó có thể thấy χ xác định duy nhất theo giá trị của nó trên {A1 , . . . , Ak }. Lập luận tương tự như trên ta suy ra được χ có đúng |A1 | · · · |Ak | = |G| đặc trưng. 4. Giả sử G là một nhóm Abel hữu hạn. Ta biết rằng χ(E) = 1 với mọi đặc trưng χ. Ta sẽ chỉ ra rằng với mọi A khác phần tử đơn vị, tồn tại một đặc trưng χ sao cho χ(A) khác 1. Thật vậy, theo (3), giả sử A có biểu diễn A = At11 · · · Atkk . Khi đó tồn tại ít nhật một chỉ số i sao cho ti > 0. Không mất tính tổng quát ta giả sử t1 > 0. Khi đó xét đặc 2pii trưng χ xác định bởi χ(A2 ) = · · · = χ(Ak ) = 1 và χ(A1 ) = e r1 . Khi đó ta có χ(A) = e 2πiti ri hiễn nhiên khác 1. 5. Các đặc trưng của một nhóm Abel G lập thành một nhóm Abel với phép nhân được xác định như sau (χ.φ)(A) = χ(A).φ(A). Phần tử đơn vị chính là đặc trưng tầm thường. Đối với một đặc trưng χ phần tử nghịch đảo của nó được xác định χ−1 (A) = χ(A−1 ). Theo nhận xét (3) nhóm các đặc trưng của G với phép toán xác định như trên có đúng |G| phần tử. Ta kí hiệu nhóm này là G. Ta biết rằng đặc trưng χ xác định trong (4), sinh ra một nhóm con cyclic của G với bậc là r1 . Tương tự, G có các nhóm con cyclic có cấp r2 , . . . , rn . Do đó dễ thấy rằng G là tổng trực tiếp của các nhóm con cyclic có cấp r1 , . . . , rn . Nói cách khác G và G đẳng cấu. Sự đẳng cấu không tự nhiên do nó phụ thuộc vào phân tích của G thành tổng trực tiếp của các nhóm con cyclic. 89 6. Cho H là một nhóm con của G. Giả sử χ là một đặc trưng của H. Một câu hỏi tự nhiên là liệu ta có thể mở rộng đặc trưng này lên G. Câu trả lời cho trường hợp này là có thể. Ta sẽ chứng minh khẳng định này bằng quy nạp theo [G : H]. Nếu [G : H] = 1 thì khẳng định hiển nhiên đúng. Giả sử ta đã chứng minh đúng với mọi G, H mà [G : H] < k. Ta sẽ chứng minh điều này cũng đúng nếu [G : H] = k. Thực vậy, do H = G nên tồn tại x ∈ G \ H. Gọi n là cấp của x + G/H trong nhóm thương G/H. Đặt t = χ(xn ). Do C∗ là nhóm chia được nên tồn tại ω sao cho ω n = t. Gọi H là nhóm con của G sinh bởi H và x. Khi đó mọi phần tử của H có thể viết dưới dạng hxa với a ∈ Z. Ta định nghĩa: χ (h ) = χ(h)ω a . Dễ dàng kiểm tra rằng định nghĩa trên không phụ thuộc vào việc chọn đại diện và đó thực sự là một đặc trưng của H . Khi đó [G : H ] < k, áp dụng giả thiết quy nạp ta có điều phải chứng minh. Chú ý rằng bằng lập luận tương tự ta có thể thấy rằng, mọi đặc trưng của H có đúng [G : H] mở rộng lên G. Ta kết thúc nhận xét (6) bằng một hệ quả quan trọng đó là dãy: 1 → G/H → G → H là một dãy khớp ngắn. 3 Tổng các đặc trưng và quan hệ trực giao Xét G là một nhóm Abel hữu hạn có cấp là h. Khi đó ta xét hai tổng quan trọng sau: S= χ(A), A∈G với χ là một đặc trưng cho trước của G, và tổng sau: T = χ(A), χ trong đó A là một phần tử cho trước và tổng được lấy trên tất cả các đặc trưng của G. Đầu tiên ta sẽ tính S. Lấy B là một phần tử bất kì của G. Khi đó ta có Sχ(B) = χ(AB). A∈G 90 Ta biết rằng ánh xạ: G → G xác đinh bởi: A → AB là một song ánh. Do đó vế phải của đẳng thức trên chính là S. Từ đó ta suy ra S(χ(B) − 1) = 0 với mọi phần tử B của G. Do đó nếu χ là một đặc trưng không tầm thường thì tồn tại B sao cho χ(B) khác 1. Nói cách khác, ta có S = 0. Trong trường hợp χ là một đặc trưng tầm thường ta χ(A) = 1 với mọi A. Từ đó suy ra S = |G|. Từ những lập luận trên ta suy ra S= |G|, nếu χ = χ1 0, nếu χ = χ1 . Bằng phương pháp tương tự, lấy φ là một đặc trưng bất kì của G. Nhân cả hai về của T với φ(A) ta suy ra được φ(A).T = T với mọi đặc trưng φ. Theo nhận xét 4 nếu A khác phần tử đơn vị thì luôn có một đặc trưng φ sao cho φ(A) = 1. Từ đó ta suy ra T = |G|, nếu A = E 0, nếu A = E. Quay lại với vấn đề ta cần xem xét. Giả sử m là một số nguyên dương. Khi đó nhóm Abel nhân tính (Z/mZ)∗ có đúng ϕ(m) phần tử, trong đó ϕ(m) là hàm Euler. Trong phần tiếp theo ta sẽ xem xét các đặc trưng của nhóm này. Tuy nhiên, trước hết ta có một nhận xét đơn giản sau. Ta có thể mở rộng miền xác định của đặc trưng của nhóm này lên toàn bộ tập hợp số nguyên như sau χ(a) = χ(A). trong đó A là một lớp nguyên tố thặng dư modulo m và chứa a. Với những số nguyên a mà gcd(a, m) > 1 ta định nghĩa χ(a) = 0. Từ những nhận xét trên, ta định nghĩa một đặc trưng theo modulo m là một hàm số học χ có những tính chất sau 1. χ(a) = χ(b) với a ≡ b (mod m). 2. χ(ab) = χ(a)χ(b) với mọi a, b. 3. χ(a) = 0 nếu gcd(a, m) > 1. 4. χ(a) = 0 nếu gcd(a, m) = 1. Có đúng ϕ(m) đặc trưng modulo m. Các đặc trưng này lập thành một nhóm Abel, đẳng cấu với nhóm các lớp thặng dư nguyên tố modulo m. Phần tử 91 đơn vị của nhóm này là đặc trưng tầm thường. Hơn nữa, theo kết quả ở trên, ta có các quan hệ trực giao như sau ϕ(m), nếu χ = χ1 0, nếu χ = χ1 . χ(n) = n (mod m) và χ(n) = χ ϕ(m), nếu n ≡ 1 mod m 0, trường hợp còn lại. Ta sẽ nêu ra một ví dụ cụ thể. Xét m = 6. Khi đó ϕ(6) = 2 Khi đó có 2 lớp đồng dư nguyên tố modulo 6 là E, A trong đó A là lớp tất cả các số nguyên đồng dư với 5 modulo 6. Như vậy ta có hai đặc trưng modulo 6. Đặc trưng thứ nhất là đặc trưng tầm thường. Đặc trưng thứ hai được xác định như sau   nếu n ≡ 1 (mod 6) 1, χ2 (n) = −1, nếu n ≡ 5 (mod 6)   0, các trường hợp còn lại. 4 Chuỗi Dirichlet và định lí Landau ∞ Một chuỗi Dirichlet là một chuỗi có dạng n=1 an ns trong đó s và các hệ số an nhận giá trị phức. Một cách tổng quát hơn, một chuỗi có dạng ∞ n=1 hoặc an , λs ∞ an e−sλn , n=1 trong đó 0 < λ1 < λ2 < · · · và λn tiến tới vô cùng khi n ra vô cùng, cũng được gọi là một chuỗi Dirichlet. Rất nhiều chuỗi Dirichlet xuất hiện trong lí ∞ thuyết số có dạng an n−s . Trong phần tiếp theo ta sẽ xem xét một số tính n=1 chất cơ bản của chuỗi dạng này. Để thuật tiện ta viết s = σ + it với σ và t là các số thực. Tương tự như trong chuỗi lũy thừa, ta có một định lí quan trọng sau đây. 92 −s Định lý 1. Giả sử chuỗi ∞ hội tụ với s = s0 , khi đó nó cũng hội n=1 an n tụ đều trong miền xác định bởi | arg(s − s0 )| ≤ π2 − θ < π2 . Chứng minh. Không mất tính tổng quát, ta có thể giả sử rằng s0 = 0. Thực vậy, ta có thể viết lại chuỗi dưới dạng an bn = s−s0 , s n n trong đó bn = an . n−s0 ∞ an hội tụ, ta có lim rn = 0 với rn = Do n→∞ n=1 v>n av . Xét M < N là các số nguyên dương. Ta có N n=M N N an rn − rn−1 = = s n ns n=M n=M rn rn − s s (n + 1) n + rM −1 rN − . s M (N + 1)s Với σ > 0 ta có 1 1 − = s (n + 1)s ns n+1 n dx ≤ |s| xs+1 Hơn nữa do lim rn = 0, với mỗi n→∞ n+1 n dx |s| = xσ+1 σ 1 1 − nσ (n + 1)σ . ta có thể tìm được n( ) sao cho với mỗi n > n( ) sao cho |rn | < . Do đó nếu ta xét M > n( ) thì ta luôn có N n=M |s| an < s n σ 1 1 − σ M (N + 1)σ + Mσ + (N + 1)σ . Do |s| ≥ σ, ta suy ra N n=M an |s| 2 |s| ≤ + σ ≤ . s σ n σM M σ Ta có nhận xét quan trọng sau 1 1 1 |s| = ≤ . = π σ cos | arg(s)| cos( 2 − θ) sin(θ) Tổng hợp lại, với mỗi s sao cho | arg(s)| ≤ N n=M an 2 < , s n sin θ π 2 −θ < π 2 ta luôn có N > M > n( ). Bất đẳng thức trên không phụ thuộc vào s, ta kết luận được chuỗi đã cho hội tụ đều trong miền | arg(s)| ≤ π2 − θ < π2 . Định lí 1 do đó được chứng minh. 93 Định lí 1 cho ta kéo theo các định lí sau. ∞ Định lý 2. Nếu chuỗi n=1 an ns hội tụ với s = s0 thì nó cũng hội tụ trên nửa mặt phẳng σ > σ0 và hội tụ đều trên mọi tập compact nằm trong nửa mặt phẳng đó. Chứng minh. Điều này được suy ra tự nhận xét là mọi tập compact trong nửa mặt phẳng nói trên đều nằm trong một miền nào đó có dạng | arg(s − s0 )| ≤ π − θ. Theo định lí 1, chuỗi Dirichlet đã cho hội tụ đều trong miền này, do 2 đó nó cũng hội tụ đều trên tập compact đang xét. Một hệ quả của định lí 2 đó là ∞ Định lý 3. Nếu chuỗi n=1 an ns hội tụ với s = s0 tới tổng f (s0 ) và f (s) là tổng chuỗi xác định trên nửa mặt phẳng σ > σ0 thì ta luôn có f (s) → f (s0 ) khi s → s0 theo mọi đường nằm trong miền | arg(s − s0 )| ≤ π2 − θ < π2 . Chứng minh. Với mọi đường cong C nằm trong một miền nào đó có dạng | arg(s − s0 )| ≤ π2 − θ thì chuỗi Dirichlet đang xét hội tụ đều trong miền này. Do giới hạn của một chuỗi hội tụ đều trong một miền compact là liên tục, ta nhận được điều phải chứng minh. Định lí 2 chỉ ra rằng miền hội tụ của chuỗi Dirichlet là một nửa mặt phẳng. Theo đó những điểm trên trục thực được chia thành hai lớp U và L với: ∞ an hội tụ}, U = {σ| ns n=1 và ∞ L = {σ| n=1 an phân kì}. ns Theo định lí 2, mỗi phần tử trong U đều lớn hơn mọi phần tử trong L. Do đó tồn tại một phần tử σ0 sao cho chuỗi hội tụ với σ > σ0 và phân kì với σ < σ0 . Tuy nhiên ta không thể kết luận được gì trong trường hợp σ = σ0 . Nếu U rỗng, ta định nghĩa σ0 = ∞, nếu L rỗng ta định nghĩa σ0 = −∞. σ0 được gọi là hoành độ hội tụ, đường thẳng σ = σ0 được gọi là đường hội tụ và nửa mặt phẳng σ > σ0 được gọi là nửa mặt phẳng hội tụ của chuỗi an Dirichlet ∞ n=1 ns . Chúng ta xét một vài ví dụ. 94 ∞ 1. Ta xét chuỗi Dirichlet n=1 n! . ns Khi đó các số hạng n! ns không hội tụ về 0 khi n ra vô cùng. Do đó chuỗi này phân kì với mọi s. Theo định nghĩa σ0 = +∞. ∞ 2. Chuỗi n=1 1 n!ns hội tụ với mọi s. Điều này dễ dàng suy ra thông qua một số đánh giá cơ bản. Do đó ta cũng suy ra được σ0 = −∞. Trước khi chứng minh đến định lí 4, ta cần một số kết quả cơ bản trong giải tích phức liên quan đến tính chỉnh hình của một chuỗi các hàm phức. Cụ thể ta có bổ đề sau. Bổ đề 1. Cho Ω là một miền trong mặt phẳng phức và {fn } là một dãy các hàm chỉnh hình trong Ω. Giả sử rằng {fn } hội tụ đều tới một hàm f trong mọi tập compact chứa trong Ω. Khi đó f cũng là hàm chỉnh hình trên Ω. (k) Hơn nữa dãy các hàm chỉnh hình {fn } cũng hội tụ đều tới f (k) trong mọi tập compact trong Ω với F (k) là đạo hàm thứ k của hàm F . Từ bổ đề trên và định lí 2, ta có được định lí quan trọng sau đây. ∞ Định lý 4. Một chuỗi Dirichlet dạng an n−s biểu diễn một hàm chỉnh n=1 hình theo s trên nửa mặt phẳng hội tụ của nó và các đạo hàm của f có thể nhận được bằng các lấy đạo hàm từng số hạng trong chuỗi. Ta có thể thấy rằng, những kết quả trong các định lí ở trên không hề đề cập tới sự hội tụ hay là tính chỉnh hình trên đường thẳng σ = σ0 . Đối với một chuỗi lũy thừa ta luôn có ít nhật một kì dị trên đường tròn hội tụ (do một chuỗi lũy thừa với bán kính hội tụ hữu hạn thì không thể thác triển thành một hàm chỉnh hình trên toàn bộ mặt phẳng phức). Tuy nhiên đối với các chuỗi Dirichlet, có thể xảy ra trường hợp là không có bất kì kì dị nào trên đường thẳng hội tụ. Chúng ta thể kết luận được gì về tính hội tụ, chỉnh hình của chuỗi tại một điểm cố định trên đường thẳng hội tụ? Ta sẽ quay lại câu hỏi này trong những phần tiếp theo. Tương tự như trong chuỗi lũy thừa, ta có khái niệm sau đây ∞ Định nghĩa 1. Chuỗi n=1 an ns ∞ được gọi là hội tuyệt đối nếu chuỗi n=1 |an | nσ hội tụ. Hoành độ của sự hội tụ tuyệt đối σ ¯ được định nghĩa là hoành độ hội tụ ∞ của chuỗi n=1 |an | . ns 95 Ta có một nhận xét đơn giản sau σ ¯ ≥ σ. Điều này được suy ra từ việc hội tụ tuyết dối kéo theo hội tụ thông thường. Trong trường hợp σ ¯ > σ0 sẽ có một dải trong mặt phẳng mà cho chuỗi hội tụ nhưng không hội tụ tuyệt đối. Dải σ0 < σ < σ ¯ được gọi là dải của hội tụ điều kiện. Ta sẽ xem xét vấn đề này thông qua ví dụ cụ thể sau đây. Xét chuỗi Dirichlet ∞ (−1)n−1 . ns n=1 Chuỗi này hội tụ với mọi s > 0 do nó là chuỗi đan dấu với các số hạng giảm dần, tiến về 0. Hiển nhiên chuỗi này không hội tụ với s < 0. Do đó σ0 = 0. Chuỗi tuyệt đối tương ứng của nó là ∞ n=1 1 . ns Chuỗi này hội tụ với s > 1 và phân kì với s < 1. Do đó σ ¯ = 1. Như vậy dải hội tụ điều kiện có độ rộng là 1. Với σ > 1 chuỗi trên hội tụ tuyệt đối và do đó ta có thể sắp xếp lại các số hạng trong chuỗi. Ta suy ra ∞ n=1 1 1 1 1 (−1)n−1 = ( s + s + · · · ) − 2( s + s + · · · ) = (1 − 21−s )ζ(s), s n 1 2 2 4 ∞ với σ > 1. Tuy nhiên ta biết rằng chuỗi n=1 (−1)n−1 ns (1) hội tụ với σ > 0 và hàm (1 − 21−s )ζ(s) là hàm chỉnh hình với s > 0 do hàm ζ(s) chỉ có một cực điểm đơn tại s = 1. Do đó đẳng thức (1) cũng đúng với σ > 0 theo thác triển giải tích. Ở trên ta đã chỉ ra rằng, dải hội tụ điều kiện của chuỗi (1) có độ rộng có độ rộng là 1. Định lí sau đây sẽ chỉ rằng độ rộng của dải hội tụ điều kiện ∞ của một chuỗi Dirichlet dạng n=1 an ns không quá 1. ∞ Định lý 5. Với một chuỗi Dirichlet n=1 an ns ta luôn có σ ¯ − σ ≤ 1. Chứng minh. Để chứng minh điều này ta chỉ cần chứng minh rằng chuỗi ∞ n=1 |an | nσ+1+ hội tụ với mọi > 0. Thật vậy, do chuỗi Dirichlet ta đang xét có hoành độ hội là σ nên ta có chuỗi ∞ 1 σ+ 2 n=1 n 96 hội tụ theo định lí 2. Do đó với mọi δ > 0, ta luôn có thể tìm được N sao cho với mọi n > N ta có | Từ đó ta suy ra chứng minh. |an | nσ+1+ < δ n1+ 2 . Do chuỗi ∞ 1 n=1 n1+ 2 |an | nσ+ 2 < δ. hội tụ, ta có điều phải Tuy nhiên, định lí trên không xảy ra với các chuỗi Dirichlet có dạng tổng −s quát hơn như ∞ với (λn ) không phải là tập hợp các số nguyên n=1 an λn dương. Các ví dụ sau đây sẽ chỉ ra điều này. 1. Chuỗi mọi s. 2. Chuỗi ∞ (−1)n−1 n=1 log ns hội tụ với σ > 0 nhưng không hội tụ tuyệt đối với ∞ (−1)n √ n=2 nlog ns hội tụ với mọi s nhưng không hội tụ tuyệt đối. Chúng ta quay trở lại với câu hỏi về tính chỉnh hình của hàm xác định −s trên đường thẳng hội tụ. Trong trường hợp bởi chuỗi Dirichlet ∞ n=1 an n các hệ số an đều không âm, ta có kết quả quan trọng sau. Định lý 6 (Landau). Nếu an ≥ 0 với mọi n ≥ 1 và σ0 hữu hạn thì giao điểm của trục thực và đường thẳng hội tụ là một kì dị của tổng f (s) của chuỗi −s Dirichlet f (s) = ∞ n=1 an n . Chứng minh. Do an ≥ 0 nên ta có σ ¯ = σ0 . Do đó, tương tự như trong lập luận ở định lí 1, ta hoàn toàn có thể giả sử σ0 = 0. Điều ta cần chứng minh tương đương với sự kiện s = 0 là một kì dị của hàm f . Giả sử ngược lại, ta giả sử rằng f chỉnh hình tại s = 0, khi đó chuỗi Taylor của f tại s = 1 có bán kính hội tụ ρ > 1. Do đó, tồn tại một số thực s < 0 sao cho chuỗi Taylor ∞ v=0 (s − 1)v (v) f (1), v! hội tụ. Theo phần trước, với σ > 0 ta có ∞ an e−s log n . f (s) = n=1 Theo định lí 4, ta có ∞ f (v) (s) = an n=1 Tại s = 1 ta có ∞ f (v) (1) = n=1 (− log n)v . ns an (− log n)v . n 97 Do đó chuỗi Taylor của f tại s = 1 là ∞ v=0 (s − 1)v v! ∞ n=1 an (− log n)v = n ∞ v=0 (1 − s)v v! ∞ n=1 an (log n)v . n Do tất cả số hạng của chuỗi đôi là các số thực không âm khi s < 0 do đó ta có thể thay đổi thứ tự lấy tổng mà giá trị của chuỗi là không đổi. Tức là chuỗi ∞ ∞ an (1 − s)v (log n)v , n v=0 v! n=1 hội tụ với s < 0. Tuy nhiên ta lại có ∞ v=0 (1 − s)v (log n)v = e(1−s) log n . v! −s hội tụ với s < 0 nào đó. Điều này trái ngược với giả Do đó chuỗi ∞ n=1 an n thiết σ0 = 0. Ta kết luận được s = 0 là một kì dị của hàm f . Chứng minh kết thúc. 5 Tích của hai chuỗi Dirichlet Tương tự như đối với chuỗi lũy thừa, ta cũng có hy vọng có thể nghiên cứu tính chất về tích của hai chuỗi Dirichlet. trước hết, ta định nghĩa tích hình −s và thức của hai chuỗi Dirichlet. Cho hai chuỗi Dirichlet dạng ∞ n=1 ak k ∞ −s b m , khi đó ta định nghĩa tích hình thức của hai chuỗi này là một n=1 m chuỗi Dirichlet có dạng n=1 cn n−s trong đó hệ số cn được xác định bởi ak b m . cn = km=n Trong trường hợp cả hai chuỗi Dirichlet ban đầu đều hội tụ tuyệt đối tại s, ta có thể nhân các số hạng và nhóm lại, khi đó ta chuỗi hình thức xác định như trên cũng hội tụ tuyệt đối và được gọi là tích của hai chuỗi Dirichlet ban đầu. Với σ > σ0 ta đặt ∞ f (s) = k=1 ak , ks ∞ g(s) = bm . ms m=1 Theo định lí 5, hàm số h(s) = f (s)g(s) có thể biểu diễn như là tích của hai chuỗi Dirichlet trong nửa mặt phẳng σ > σ0 + 1. Định lí sau đây sẽ chỉ ra rằng, sự biểu diễn như vậy là duy nhất. Trước hết ta cần một kết quả quen thuộc sau trong giải tích phức. 98 Bổ đề 2. Cho Ω là một miền trong mặt phẳng phức và f là một hàm chỉnh hình trên Ω. Nếu tồn tại một tập S mà có ít nhất một điểm tụ trong Ω và f = 0 trên S thì f = 0 trên Ω. ∞ bn an Định lý 7. Nếu hai chuỗi ∞ n=1 ns cùng hội tụ trong nửa mặt n=1 ns và phẳng và các hàm tổng bằng nhau trong một tập mở không rỗng chứa trong nửa mặt phẳng đó thì an = bn với mọi n ≥ 1. an −bn Chứng minh. Xét chuỗi Dirichlet ∞ n=1 ns . Chuỗi này như ta đã biết hội tụ trong nửa mặt phẳng σ > σ0 và tổng chuỗi là một hàm chỉnh hình trên nửa mặt phẳng này. Theo giả thiết thì hàm tổng này bằng 0 trên một tập mở, nằm trong nửa mặt phẳng đang xét, theo bổ đề trên, ta suy ra được hàm tổng xác định ở trên bằng 0 trên nửa mặt phẳng σ > σ0 . Gọi M là chỉ số đầu tiên mà an = bn . Đặt cn = an − bn . Khi đó với σ > σ0 ta có: ∞ ∞ cn cn = = 0. s n ns n=1 n=M Hay là ∞ cM cn =− . σ M nσ n=M +1 Do chuỗi trên hội tụ đều trên nửa mặt phẳng σ > σ0 + 2 nên nếu ta cho σ → ∞, ta suy ra ngay được cM = 0. Điều này trái ngược với giả thiết cM = 0. Ta có điều phải chứng minh. 6 Định lí Dirichlet Ở các phần trước ta đã chuẩn bị các kiến thức cần thiết, ở phần này ta sẽ áp dụng những kết quả về đặc trưng và chuỗi Dirichlet dạng ∞ n=1 χ(n) , n (1) trong đó χ là một đặc trưng modulo m để chứng minh định lí Dirichlet về số nguyên tố trong một cấp số cộng. Số đặc trưng modulo m là ϕ(m) do đó có đúng ϕ(m) chuỗi Dirichlet dạng trên. Do |χ(n)| ≤ 1 nên các chuỗi trên hội tụ trên nửa mặt phẳng σ > 1. Ta kí hiệu hàm tổng tương ứng với chuỗi Dirichlet xác định bởi đặc trưng χ là L(s, χ). Các hàm này được gọi là các L-hàm Dirichlet. Ta sẽ xem xét các tính chất của các hàm này. Ta xét hai trường hợp riêng biệt. 99 1. Nếu χ = χ1 chuỗi Dirichlet xác định bởi (1) hội tụ trên nửa mặt phẳng σ > 0 do n≤x χ(n) là bị chặn. Chi tiết của nhận xét này như sau: Ta phân hoạch các số nguyên từ 1 tới [x] vào các lớp thặng dư modulo m. Ta viết [x] = mq + r với 0 ≤ r < m theo thuật toán Euclide. Từ đó ta có   [x] χ(n) = n=1 n≤x mq+r mq m +··· + χ(n) =  n=1 χ(n).  χ(n) + mq+1 m(q−1)+1 Theo quan hệ trực giao ở phần 3, ta nhận được mq+r χ(n). χ(n) = mq+1 n≤x Từ đó ta suy ra được mq+r | |≤ Đặt rk = M n=1 k n=1 |χ(n)| ≤ r < m. n=mq+1 n≤x χ(n). Khi đó |rk | < m với mọi k. Khi đó ta có χ(n) = ns M −1 n=1 rn − rn−1 = ns M −1 rn ( n=1 1 1 rM − ) + s. s s n (n + 1) M Do đó với σ > 0 ta nhận được M n=1 χ(n) ≤ nσ M −1 |rn | n=1 1 1 rM − + σ ≤ σ σ n (n + 1) M M m 1− n=1 1 1 +m σ = m. σ M M Do đó chuỗi Dirichlet trên hội tụ với s = σ > 0 và do đó theo định lí ở phần trước, chuỗi này hội tụ trên nửa mặt phẳng σ > 0. Vơi σ < 0 chuỗi này hiển nhiên phân kì. Do đó hoành độ hội tụ là σ0 = 0 và hoành độ của hội tụ tuyệt đối là σ ¯ = 1. Do đó theo định lí 4, L(s, χ) với χ = χ1 là một hàm chỉnh hình trên nửa mặt phẳng σ > 0. 2. χ = χ1 , ta sử dụng đồng nhất thức Euler ∞ ζ(s) = n=1 1 = ns 100 (1 − p 1 −1 ) , σ > 1. ps Do các đặc trưng là nhân tính chặt, theo định lí 5 ở chương VII, ta có: ∞ L(s, χ) = n=1 χ(n) = ns (1 − p χ(p) −1 ) , ps σ > 1. Từ đó ta có nhận xét đơn giản đó là L(s, χ) = 0 với mọi σ > 1. Sử dụng đẳng thức ở trên ta suy ra ngay được (1 − L(s, χ1 ) = pm 1 −1 ) = ps (1 − p 1 −1 ) · ps (1 − p|m 1 ). ps Nói cách khác ta có (1 − p−s ), L(s, χ1 ) = ζ(s) · (σ > 1). p|m Như đã chứng minh ở phần trước, ζ(s) là một hàm phân hình trên nửa mặt phẳng σ > 0, có một cực điểm đơn tại 1 như là kì dị duy nhất. Do đó L(s, χ1 ) chỉnh hình trên σ > 0 trừ tại điểm s = 1 mà ở đó nó có . cực điểm đơn với thặng dư là p|m (1 − p−1 ) = ϕ(m) m Để chứng minh định lí Dirichlet, ta cần bổ đề sau đây. Bổ đề 3. Nếu χ = χ1 ta có L(1, χ) = 0. Trước khi chứng minh bổ đề này, ta đưa ra một số lưu ý về hàm Logarith. Với |a| < 1 ta định nghĩa 1 log( )= 1−a ∞ n=1 an . n 1 1 Dễ dàng kiểm tra hàm log này thỏa mãn điều kiện elog( 1−a ) = 1−a với mọi |a| < 1. Với sự kiện này ta có thể chứng minh định lí trên như sau. Chứng minh. Để chứng minh điều này ta chỉ cần chứng mình tích P (s) = L(s, χ), χ không phải là hàm chỉnh hình trên nửa mặt phẳng σ > 0. Thực vậy, nếu P (s) là một hàm chỉnh hình trên nửa mặt phẳng này, cực điểm đơn của L(s, χ1 ) tại s = 1 phải bị triệt tiêu bởi không điểm s = 1 của một L-hàm nào đó. Do 101 vậy, để chứng minh bổ đề đã cho ta chỉ cần chứng minh rằng P (s) không là hàm chỉnh hình trên nửa mặt phẳng σ > 0. Với σ > 1 ta có |χ(p)p−s | ≤ p−σ < 1, do đó ta định nghĩa log(1 − χ(p) −1 ) = ps k χ(pk ) . kpkσ Do đó hàm log L(s, χ) xác định duy nhất trên nửa mặt phẳng σ > 1 và cho bởi χ(pk ) log L(s, χ) = . kpks p,k với p chạy trong tập số nguyên tố và k chạy trong tập số tự nhiên. Chuỗi kép này hội tụ tuyệt đối trên nửa mặt phẳng σ > 1 và hơn nữa elog L(s,χ) = L(s, χ). Nếu ta lấy tổng trên tất cả các đặc trưng χ modulo m ta có Q(s) = log P (s) = log L(s, χ) = χ χ p,k χ(pk ) . kpks Do chỉ có hữu hạn đặc trưng χ nên ta có thể thay đổi thứ tự lấy tổng và nhận được 1 χ(pk ). Q(s) = ks kp χ p,k Theo quan hệ trực giao ta có χ(a) = χ ϕ(m), nếu a ≡ 1 (mod m) 0, trường hợp còn lại . Do đó ta có Q(s) = ϕ(m) pk ≡1 (mod m) 1 . kpks Nếu ta định nghĩa an = ϕ(m) , k 0, nếu n = pk ≡ 1 (mod m) trường hợp còn lại. thì ta có ∞ Q(s) = n=1 102 an , ns trong đó các hệ số an đều không âm. Chuỗi này hội tụ với σ > 1. Để tìm hoành độ hội tụ, gọi p là một số nguyên tố mà p |m. Theo định lí Euluer, ta có ph ≡ 1( mod m) với h = ϕ(m). Nếu ta chỉ xét chuỗi này với s thực và chỉ xét số hạng mà k = h thì ta có Q(s) > pm 1 = phs p 1 − phs p|m 1 . phs Chuỗi p p1 phân kì và p|m p1 hữu hạn nên ta chuỗi xác định ở trên phân kì với s = h1 Do đó nếu α là hoành độ của hội tụ thì ta có α ≥ h1 . Tuy nhiên ta lại có Q(s)s P (s) = eQ(s) = 1 + Q(s) + + ··· (∗) 2! Ta biết rằng tích của hai chuỗi Dirichlet với hệ số không âm lại là một chuỗi Dirichlet với hệ số không âm và chuỗi này hội tụ trên giao của hai nửa mặt phẳng hội tụ của hai chuỗi ban đầu. Do đó tất cả các lũy thừa Qn (s) hội tụ tuyệt đối. Do đó chuỗi Dirichlet xác định bởi (*) cũng có hệ số không âm. Do vậy nếu chuỗi Dirichlet Q(s) hội tụ, thì chuỗi Dirichlet của P (s) cũng vậy. Ngược lại nếu chuỗi Dirichlet của P (s) hội tụ với số thực s nào đó thì chuỗi Dirichlet của Q(s) cũng vậy do các hệ số của nó đều không âm với giá trị tại s. Từ nhận xét trên, ta biết rằng chuỗi Dirichlet của Q(s) và P (s) có cùng hoành độ hội tụ σ0 = α. Theo định lí Landau s = α là một kì dị của P (s). Tuy nhiên ta biết rằng α ≥ h1 > 0. Do đó hàm P (s) không chỉnh hình trên toàn bộ nửa mặt phẳng σ > 0. Bổ đề do đó được chứng minh. Trên đây là chứng minh dựa theo cuốn [1]. Trong phần tiếp theo ta sẽ chứng bày một chứng minh khác, có phần đơn giản hơn chứng minh trên. Chứng minh này dựa chủ yếu và quyển của [2] Ta vẫn đặt P (s) = χ L(s, χ). Để thuận tiện, trước hết ta đưa ra một số khái niệm cơ bản sau đây. Nếu p không chia hết m, ta kí hiệu p¯ là ảnh của p trong (Z/mZ)∗ . Gọi f (p) . Khi đó ta có là bậc của p¯ trong nhóm này và g(p) = ϕ(m) f (p) Bổ đề 4. Với p m ta có đẳng thức: (1 − χ(p)T ) = (1 − T f (p )g(p) . χ∈G với G là nhóm các đặc trưng của (Z/mZ)∗ . Chứng minh. Thực vậy, gọi W là tập hợp tất cả các căn f (p) của đơn vị. Khi đó ta có (1 − wT ) = 1 − T f (p) . w∈W 103 Ta biết rằng với mỗi w ∈ W tồn tại duy nhất một đặc trưng của nhóm cyclic = g(p) sinh bởi p¯ sao cho χ(¯ p) = w. Theo nhận xét 5 ở bài 2, có đúng ϕ(m) f (p) mở rộng của đặc trưng này lên G. Do đó mỗi số hạng trong tích trên xuất hiện đúng g(p) lần. Từ đó ta dễ dàng có điều phải chứng minh. Áp dụng bổ đề trên và công thức tích của L(s, χ) ta có một đẳng thức quan trọng sau 1 P (s) = . (1 − p−sf (p) )g(p) pm Đây cũng là một chuỗi Dirichlet với hệ số dương và hội tụ trên nửa mặt phẳng σ > 1. Sử dụng khai triển Taylor, ta có 1 (1 − p−sf (p) )g(p) = (1 + p−sf (p) + p−2sf (p) + · · · )g(p) . Chuỗi này hiển nhiên chặn chuỗi 1 + ps−ϕ(m) + p−2sϕ(m) + · · · . Từ đó P (s) có tất cả các hệ số lớn hơn hệ số tương ứng của chuỗi n−sϕ(m) . (n,m)=1 1 Chuỗi trên phân kì với s = ϕ(m) , do đó P (s) không thể là hàm chỉnh hình trên toàn bộ mặt phẳng σ > 0. Theo nhận xét của chứng minh 1, ta suy ra L(1, χ) = 0 với mọi χ = χ1 . Trước khi đến với định lí ta cần một số đánh giá cơ bản. Ta biết rằng nếu χ = χ1 thì log log L(s, χ) = p 1 = 1 − χ(p)p−s Ta chia tổng này thành hai phần fχ (s) = pm và Fχ (s) = χ(p) , ps χ(p)n . ns np p,n≥2 104 n,p χ(p)n . npns Với σ > 1 2 ta có χ(p)n 1 ≤ 2σ . ns np np Do đó nếu Fχ (s) là hàm chỉnh hình với mọi s > 12 . Do log L(s, χ) là hàm chỉnh hình trên σ > 0. Do đó ta suy ra được fχ (s) bị chặn khi s → 1 với mọi χ = χ1 . Ta sẽ áp dụng những kiến thức vừa trình bày để chứng minh định lí Dirichlet đưa ra ở lời tựa. Tuy nhiên ta sẽ chứng minh một phiên bản tốt hơn. Trước hết ta đưa ra khái niệm trù mật theo kiểu Dirichlet. Giả sử P là tập tất cả các số nguyên tố. Ta biết rằng 1 1 ). ∼ log( ps s−1 p Gọi A là một tập con của P , ta nói A có độ trù mật Dirichlet bằng k nếu lim s→1+ p∈A p−s 1 log s−1 = k. Định lí của Dirichlet phát biểu rằng Định lý 8 (Dirihlet). Cho m ≥ 1 và a là số nguyên sao cho (a, m) = 1. Gọi Pa là tập tất cả các số nguyên tố sao cho p ≡ a mod m. Khi đó Pa có độ 1 trù mật ϕ(m) . Chứng minh. Ta xét hàm ga (s) = p∈Pa Khi đó ta có ga (s) = 1 ϕ(m) χ(a)−1 fχ (s). χ Thực vậy ta viết tường minh các số hạng χ χ(a)−1 fχ (s) dưới dạng χ(a−1 )χ(p) p−s . ( pm 1 . ps χ Ta lại có χ(a−1 )χ(p) = χ(a−1 p). 105 Theo quan hệ trực giao ta có χ(a−1 p) = χ ϕ(m), nếu p ≡ a (mod m) 0, trường hợp còn lại. Từ đó ta dễ dàng suy ra tổng trên chính bằng ϕ(m)ga (s). Theo các nhận xét 1 ) khi s → 1 và với mọi χ = χ1 thì fχ (s) bị chặn khi ở trên fχ1 (s) ∼ log( s−1 1 ) ta nhận được s → 1. Bằng cách chia cả hai vế cho log( s−1 ga (s) ∼ 1 1 log( ). ϕ(m) s−1 Theo định nghĩa, hiển nhiên Pa có độ trù mật theo kiểu Dirichlet là 1 . ϕ(m) TÀI LIỆU THAM KHẢO [1] K. Chandrasekharan, Introduction to Analytic Number Theory, Springer, 1968. [2] J. P. Serre, A course in Arithmetic, Springer, 1996. 106 Định lí số nguyên tố Người trình bày: Phạm Ngọc Hoàng Minh Nội dung bài này dựa theo chương XI [1]. Chúng ta đã chứng minh ở chương VII rằng log(x)π(x) log(x)π(x) ≤ 1 ≤ lim sup . lim inf x→∞ x x x→∞ Mục đích của chương này là chỉ ra tồn tại giới hạn lim x→∞ log(x)π(x) . x Và từ đó ta suy ra định lý số nguyên tố log(x)π(x) = 1. x→∞ x lim Định lý 1 (Hadamard-De La Vallée Poussin). Cho t = 0. Khi đó, ζ(1+it) = 0. Chứng minh. Khi σ > 1, như đã chứng minh trong bài trước, ta có 1 . 1 − 1/ps ζ(s) = p Lấy logarithm hai vế với Re(s) > 1, thì như trong chương X, ta suy ra log ζ(s) = p,m với 1 = mpms ∞ n=1 cn , ns   1 , nếu n = pm cn = m 0, nếu n = pm . 107 Khi đó, do cn cn = σ cos(t log(n)) + i sin(t log(n)) , nên s n n ∞ log |ζ(s)| = Re n=1 cn ns = cn cos(t log(n)). nσ Vậy, ∞ 3 4 log |ζ (σ)ζ (σ+it)ζ(σ+2it)| = n=1 cn (3 + 4 cos(t log(n)) + cos(2t log(n))) ≥ 0. nσ Từ đó, ta suy ra rằng với mọi σ > 1 và t là số thực thì |ζ 3 (σ)ζ 4 (σ + it)ζ(σ + 2it)| ≥ 1. Bây giờ, ta giả sử tồn tại t0 = 0, sao cho ζ(1 + it0 ) = 0. Khi đó, do hàm ζ(s) chỉ có 1 cực duy nhất tại s=1, và đó là cực đơn, ta suy ra rằng nếu ta cho σ → 1 thì |ζ 3 (σ)ζ 4 (σ + it0 )ζ(σ + 2it0 )| −→ 0. Từ đó và bất đẳng thức đã chứng minh ở trên, ta suy ra điều mâu thuẫn. Vì vậy, điều giả sử là sai và ta kết thúc chứng minh định lý. Định lý 2 (Wiener-Ikehara). Cho A(x) là 1 hàm không tăng, đi từ [0, ∞) vào [0, ∞). Giả sử tích phân ∞ A(x)e−sx dx 0 hội tụ về f (s), với mọi s = σ + it, mà σ > 1. Trong đó, f (s) là hàm giải tích trên nửa mặt phẳng Re(s) ≥ 1 ngoại trừ tại điểm cực duy nhất s = 1. Cực đó là cực đơn, với thặng dư bằng 1. Khi đó, lim A(x)e−x = 1. x→∞ Chứng minh. Đặt B(x) = A(x)e−x , ta sẽ chứng minh định lí trên thông qua 2 bước. Trước tiên, ta chứng minh rằng λy v sin2 v dv = π. lim B y− y→∞ −∞ λ v2 Sau đó, ta dùng đẳng thức trên để chỉ ra rằng lim B(x) = 1. x→∞ 108 1 . Khi đó, do tính chất của f (s), thì g(s) s−1 là hàm giải tích trên nửa mặt phẳng Re(s) ≥ 1. Vì vậy, nếu Re(s) > 1 thì BƯỚC 1: Ta đặt g(s) = f (s) − ∞ A(x)e−x − 1 ex(1−s) dx. g(s) = 0 Bây giờ, ta lại xét 1 2 2λ g(1 + ε + it) 1 − −2λ |t| 2λ eiyt dt = 1 2 2λ 1− −2λ |t| 2λ ∞ (B(x) − 1)e−(ε+it)x dx dt. eiyt 0 Ta đặt 1− ψ(t, x) = |t| 2λ eiyt (B(x) − 1)e−(ε+it)x . Vì A(x)e−sx ∈ L1 ((0, ∞)) khi Re(s) > 1, nên A(x) là hàm đo được trên (0, ∞). Từ đó, ta suy ra ψ cũng là hàm đo được. Hơn nữa, |t| ||B(x) − 1|e−εx , 2λ |ψ(t, x)| = |1 − nên khi ta đặt ∞ |ψ(t, x)|dt ϕ(t) = 0 thì ϕ(t) = |1 − Mà ta lại có A(x)e −(1+ε)x ∞ |t| | 2λ và e |A(x)e−(1+ε)x − e−εx |dx. 0 −εx đều nằm trong L1 ((0, ∞)) nên ϕ(t) < ∞, và vì thế, 2λ |ϕ(t)|dt < ∞. −2λ Vậy theo định lý Fubini, ta suy ra ψ(t, x) ∈ L1 ((−2λ, 2λ) × (0, ∞)). Do đó, cũng theo định lý Fubini, ta có thể đổi thứ tự tính tích phân cho ψ mà vẫn thu được cùng kết quả, nghĩa là 2λ 1− −2λ ∞ |t| 2λ ∞ 0 2λ 1− = −2λ 0 (B(x) − 1)e−(ε+it)x dxdt eiyt |t| 2λ eity (B(x) − 1)e−(ε+it)x dtdx ∞ (B(x) − 1)e−εx =2 0 sin2 (λ)(y − x) dx. λ(y − x)2 109 Vậy 1 2 2λ g(1 + ε + it) 1 − −2λ ∞ |t| 2λ (B(x) − 1)e−εx eiyt dt = 0 sin2 (λ(y − x)) dx. λ(y − x)2 Chọn ε trong đẳng thức trên lần lượt bằng với εn = n1 với mọi n nguyên dương. Khi đó, cho n −→ ∞, sử dụng định lý hội tụ tuyệt đối, và chú ý sự hội tụ đều của g(s) ta có 2λ g(1 + εn + it) 1 − −2λ và ∞ e−εn x 0 2λ |t| 2λ eiyt dt −→ g(1 + it) 1 − −2λ sin2 (λ(y − x)) dx −→ λ(y − x)2 ∞ 0 |t| 2λ eiyt dt, sin2 (λ(y − x)) dx. λ(y − x)2 Cũng là cho n −→ ∞, nhưng lần này sử dụng định lí hội tụ đơn điệu, ta có ∞ B(x)e 0 ∞ 2 (λ(y − x)) dx −→ λ(y − x)2 −εn x sin 0 sin2 (λ(y − x)) B(x) dx. λ(y − x)2 Từ các sự hội tụ trên, ta suy ra ∞ B(x) 0 ∞ sin2 (λ(y − x)) dx = λ(y − x)2 0 2λ sin2 (λ(y − x)) 1 dx+ 2 λ(y − x) 2 g(1 + it) 1 − −2λ Cho y → ∞, do bổ đề Riemann-Lebesgue, ta thu được 2λ g(1 + it) 1 − −2λ |t| 2λ eiyt dt −→ 0. Thêm vào đó, ∞ lim y→∞ 0 sin2 (λ(y − x)) dx = lim y→∞ λ(y − x)2 λy −∞ sin2 v dv = v2 +∞ −∞ sin2 v dv = π. v Vậy, λy B y− lim y→∞ −∞ v sin2 v dv = π. λ v2 Ta kết thúc chứng minh bước 1. BƯỚC 2: Để chứng minh lim B(x) = 1, ta sẽ chứng 2 điều sau x→∞ lim sup B(x) ≤ 1, x→∞ 110 |t| 2λ eiyt dt. và lim inf B(x) ≥ 1. x→∞ a , ta có λ Cho a > 0 và λ > 0. Khi đó, với mọi y > a B y− −a λy v sin2 v dv ≤ λ v2 B y− −∞ v sin2 v dv. λ v2 Ta lại có A(x) = B(x)ex là hàm không giảm nên B y− v y−v/λ a y−a/λ e ≤B y− e , λ λ trong đó, v ∈ [−a, a]. Từ đó ta suy ra rằng với mọi v ∈ [−a, a] thì B y− a −2a/λ a v−a/λ v e ≤B y− e ≤B y− . λ λ λ Thế vào bất đẳng thức tích phân ở trên, ta được B y− a −2a/λ e λ a −a sin2 v dv ≤ v2 λy B y− −∞ v sin2 v dv. λ v2 Lấy lim sup của bất đẳng thức trên, ta suy ra y→∞ lim sup B y − y→∞ a a λ e−2a/λ −a sin2 v dv ≤ π. v2 Bây giờ, ta lần lượt cho λ → ∞ trước, rồi cho a → ∞ thì ta sẽ thu được π lim sup B(x) ≤ π, x→∞ nghĩa là lim sup B(x) ≤ 1. x→∞ Ta sẽ sử dụng kết quả trên để chứng minh lim inf B(x) ≥ 1. Thật vậy, do x→∞ lim B(x) ≤ 1, B(x) ≥ 0, và B(x) = A(x)e−x , với A(x) và e−x là các hàm x→∞ đơn điệu, ta suy ra rằng tồn tại c là số thực dương sao cho |B(x)| ≤ c, với a mọi x. Khi đó, với a > 0, λ > 0, và y > , ta có λ λy v sin2 v B y− dv ≤ c λ v2 −∞ −a ∞ + −∞ a 111 sin2 v dv + v2 a B y− −a v sin2 v dv. λ v2 Sử dụng tính không giảm của A(x) = B(x)ex , ta lại suy ra rằng với mọi v ∈ [−a, a] thì v a B y− ≤ e2a/λ B y + . λ λ Do đó, a v sin2 v a B y− dv ≤ e2a/λ B y + 2 λ v v −a a −a sin2 v dv. v2 Vậy λy B y− −∞ ∞ −a v sin2 v dv ≤ c λ v2 sin2 v a dv +e2a/λ B y + 2 v v + −∞ a a −a Lại lần lượt lấy lim inf , sau đó cho λ → ∞ và cuối cùng là cho a → ∞, ta y→∞ thu được π ≤ π lim inf B(x), x→∞ hay là 1 ≤ lim inf B(x). x→∞ Vậy ta phải có 1 ≤ lim inf B(x) ≤ lim sup B(x) ≤ 1. x→∞ x→∞ Từ đó ta có được kết quả mong muốn lim B(x) = 1, và ta kết thúc chứng x→∞ minh. π(x) log(x) = 1. x→∞ x Định lý 3 (Định lý số nguyên tố). Ta có lim Chứng minh. Như đã nói ở bài trước, khi Re(s) > 1 thì ∞ −ζ (s) = sζ(s) 1 ψ(u) du. us+1 Đổi biến u = ex , ta có −ζ (s) = sζ(s) ∞ ψ(ex )e−sx dx, 0 ở đây s = σ +it với σ > 1. Khi đó, đặt A(x) = ψ(ex ) thì A : [0, ∞) −→ [0, ∞) là hàm tăng. Hơn nữa, ζ(s) phân hình trên nửa mặt phẳng Re(s) > 0 với cực duy nhất tại s = 1. Đó là cực đơn, với thặng dư bằng 1. Kết hợp với 112 sin2 v dv. v2 định lí 1 về tính không triệt tiêu trên trục σ = 1 trừ tại s = 1, ta suy ra −ζ (s) f (s) = là hàm giải tích trên nửa mặt phẳng Re(s) ≥ 1 trừ cực duy sζ(s) nhất tại s = 1. Đó là cực đơn với thặng dư bằng 1. Vậy áp dụng định lý 2, ψ(ex ) ψ(x) ta suy ra ∼ 1. Khi đó, do liên hệ tại giới hạn ở ∞ giữa ∼ 1 hay x e x ψ(x) π(x) log(x) π(x) log(x) và đã chứng minh ở bài trước, ta suy ra ∼ 1. x x x Ta kết thúc chứng minh. TÀI LIỆU THAM KHẢO [1] K. Chandrasekharan, Introduction to Analytic Number Theory, Springer, 1968. 113 [...]... đánh giá số điểm nguyên trong miền với thể tích của miền (a) Thiết lập song ánh: Để xác định R(N ), chúng ta cần ước lượng số điểm nguyên nằm trong và trên đường tròn x2 + y 2 = N Đây là một bài toán đếm, vì vậy một cách tiếp cận tự nhiên là thiết lập một song ánh Ta chú ý rằng mỗi điểm nguyên ứng với góc tây nam của đúng một hình vuông đơn vị Theo cách đó ta có thể gán cho mỗi điểm nguyên với một hình... n) = (m )g(1, n) n Vậy ta luôn có (17) Theo Định lý 2 ta có πi πi 1 √ g(1, n) = e 4 (1−n) g(−n, 1) = e 4 (1−n) n ⇒ g(1, n) = √ (do g(−n, 1) = 1) πi n e 4 (1−n) Thay vào (17) ta có √1 (m n ) = ne πi (n−1) 4 g(m, n) Bây giờ ta sẽ chứng minh Định lý 1 (luật thuận nghịch Gauss) Trong Định lý 3 thay m = −1 ta có √1 ( −1 n ) = ne πi (n−1) 4 g(−1, n) Mà theo Định lý 2 thì πi πi 1 √ g(−1, n) = e 4 (n−1) g(−n,... số áp dụng 2 Chúng ta có công thức tính ( p ) như sau: Định lý 4 Cho p là một số nguyên tố lẻ, khi đó ta có p2 −1 2 = (−1) 8 p (19) Hay nói cách khác, 2 = p +1, −1, nếu p = ±1(mod 8), nếu p = ±3(mod 8) Chứng minh Theo Định lý 3 ta có ( p2 ) = √1p e 18 πi (p−1) 4 g(2, p), và theo Định lý 2 thì πi 1 1 √ g(2, p) = e 4 (1−2p) √ g(−p, 2), p 2 mà theo định nghĩa của g(m, n) ta có g(−p, 2) = 1 + e πip 2 Như... nói trên Từ đó định lí được chứng minh TÀI LIỆU THAM KHẢO [1] Victor Shoup, A Computational Introduction to Number Theory and Algebra, New York 2008 29 Hàm số học và điểm nguyên Người trình bày: Trần Thị Hương Giang Bài này đưa ra các ước lượng của các hàm số học Tài liệu tham khảo là chương VI [1] 1 Một Số Khái Niệm Định nghĩa 1 (Hàm tổng) Nếu f là một hàm số học thì hàm tổng của f được định nghĩa là... là hai số nguyên khác không, khi đó 1 |n| trong đó sgn(r) = 1 πi g(m, n) = e 4 (1−|mn|) sgn(mn) r |r| |m| g(−n, m) (2) nếu r = 0, và sgn(r) = 0 nếu r = 0 Chứng minh Xét hàm số 2 eπiτ u +2πiXu , Φ(u) = Φ(u, X, τ ) = 2πiu e −1 (3) với u là biến số phức, các tham số X ∈ C và τ ∈ C : Re τ > 0, và tích phân Φ(u)du với C là đường thẳng trong mặt phẳng phức đi qua điểm u = 12 và C hợp với trục số thực một góc... sau Định lí 1 Cho p, q là các số nguyên tố lẻ Khi đó ta có p q = (−1) p−1 q−1 2 2 q p Ngoài ra nếu p là một số nguyên tố lẻ thì 2 p = (−1) p2 −1 8 Chứng minh Ta trước hết chứng minh cho trường hợp p và q là các số nguyên tố lẻ Gọi ω là một nghiệm của đa thức X p − 1 trong Fq Khi đó với mỗi x ∈ Fp ta có thể thấy ω x xác định và không phụ thuộc vào cách chọn đại diện của x trong Fp Ta xét tổng Gauss... = 2 2 (q−1)/2 l=1 lp + q (p−1)/2 l=1 kq p Xét tập hợp {kq − lp | k = 1, , q−1 p−1 ; l = 1, , } 2 2 gồm p−1 q−1 số nguyên khác không Gọi S1 là số các số nguyên dương và S2 2 2 là số các số nguyên âm trong tập hợp đó, như vậy S1 + S2 = p−1 q−1 2 2 Mặt khác, S1 là số các cặp số nguyên (k, l) thỏa mãn các điều kiện 1 ≤ k ≤ p−1 , 1 ≤ l ≤ q−1 và kq − lp > 0 Với mỗi k cho trước thỏa 1 ≤ k ≤ p−1 thì... p2 , 2 do đó 1 + x2 + y 2 = mp, 0 < m < p Định lý 4 (Lagrange) Mọi số nguyên dương n đều biểu diễn được thành tổng của bốn bình phương 7 Chứng minh Ta có 1 ∈ S4 vì 1 = 12 + 02 + 02 + 02 nên ta giả sử n > 1 Do n > 1 là tích của các số nguyên tố, và 2 = 12 + 12 + 02 + 02 nên 2 ∈ S4 Do S4 đóng với phép nhân nên để chứng minh định lý ta chỉ cần chứng minh mọi số nguyên tố lẻ p đều biễu diễn được thành... số học 2.1 Hàm số dàn điểm r(n) 1 Bậc của hàm r(n) Định lý 1 lim inf r(n) = 0 n→∞ 30 Chứng minh Ta có lim inf r(n) = lim inf r(n) ≤ lim n→∞ N →∞ n≥N inf N →∞ 4k+3≥N r(4k + 3) = 0, vì r(4k + 3) = 0 với mọi k ∈ Z Mặt khác, vì r(n) ≥ 0 với mọi số nguyên dương n, ta có lim inf n→∞ r(n) ≥ 0 Như vậy chứng tỏ lim inf n→∞ r(n) = 0 Định lý 2 r(n) = O(n ) với mọi > 0 2 Bậc của hàm tổng R(n) Định lý 3 (Gauss) √... 20 Một chứng minh cho luật thuận nghịch bậc hai Người trình bày: Nguyễn Thọ Tùng Luật thặng nghịch bậc hai là một trong những kết quả vô cùng đẹp đẽ của số học Có nhiều cách chứng minh khác nhau cho định lý này Mục đích của bài viết nhỏ này là đưa ra một chứng minh cho luật thặng nghịch bậc hai dựa vào tổng Gauss Chứng minh này chủ yếu dựa vào [1] Luật thuật nghịch bậc hai được phát biểu như sau Định ... trình lý thuyết số hè 2012 Viện nghiên cứu cao cấp toán chia thành hai hoạt động Một hoạt động lớp học định lý kinh điển lý thuyết số giải tích Tài liệu học sách "Nhập môn lý thuyết số giải tích" ... nguyên 30 Định lý Chebyshev phân bố số nguyên tố 47 Định lý Weyl phân bố định lý Kronecker 67 Định lí Minkowski điểm nguyên tập lồi 77 Định lí Dirichlet số nguyên tố cấp số cộng 86 10 Định lí số nguyên... 28 số hoàn hảo Số hoàn hảo có liên quan chặt chẽ với số Mersenne Định nghĩa (Số Mersenne) Số Mersenne số nguyên có dạng 2n − Nếu số nguyên tố, ta gọi số nguyên tố Mersenne Quan hệ số hoàn hảo số

Ngày đăng: 12/10/2015, 09:09

Từ khóa liên quan

Tài liệu cùng người dùng

Tài liệu liên quan